Sie sind auf Seite 1von 39

*** AIRWAY ***

===== 1/115 Airway


Which of the following interventions is MOST appropriate when treating an unresp
onsive adult with a severe foreign body airway obstruction?
A: Abdominal thrusts, finger sweeps of the mouth, and translaryngeal jet ven
tilation
B: Subdiaphragmatic thrusts, oropharyngeal visualization, and Combitube inse
rtion
C: Back slaps, visualization of the oropharynx, finger sweeps, and chest com
pressions
D: Chest compressions, laryngoscopy and use of Magill forceps, and cricothyr
otomy
Reason:
Patients with a severe (complete) foreign body airway obstruction (FBAO) are una
ble to move any air; therefore, rapid intervention is crucial to their survival.
In the unresponsive patientregardless of ageinitial management for a severe FBAO
involves chest compressions, visualization of the mouth (finger sweeps are indic
ated only if you can see the object), and attempts to ventilate. If a few cycles
of basic interventions are unsuccessful, you should proceed to advanced techniq
ues; visualize the upper airway with a laryngoscope and remove the object with M
agill forceps if you can see it. If this technique is unsuccessful, resume basic
interventions and perform a surgical cricothyrotomy. Needle cricothyrotomy and
translaryngeal jet ventilation is contraindicated in the presence of a severe up
per airway obstruction because the jet ventilator does not allow for exhalation.
Subdiaphragmatic (abdominal) thrusts are indicated for a responsive adult or ch
ild with a severe FBAO. Back slaps and chest thrusts are indicated for a respons
ive infant with a severe FBAO. Insertion of a Combitube or other esophageal airw
ay device would be of no benefit; obturation of the esophagus will not help the
patient whose trachea is obstructed.
You selected D; This is correct!
=====2/115 Airway
A 56-year-old woman presents with acute respiratory distress. She is confused, h
as cyanosis around her mouth, and can only speak in two-word sentences. You shou
ld:
A: administer a beta-2 agonist via nebulizer.
B: assist her ventilations with a bag-mask device.
C: perform immediate nasotracheal intubation.
D: give high-flow oxygen via nonrebreathing mask.
Reason:
This patient is not ventilating adequately, as evidenced by the cyanosis around
her mouth (perioral cyanosis), her state of confusion (indicates cerebral hypoxi
a), and her inability to speak more than two words at a time (two-word dyspnea).
She needs some form of positive-pressure ventilation assistance, such as what i
s provided with a bag-mask device. Passive oxygenation (ie, nonrebreathing mask)
is of little or no benefit if the patient is not ventilating adequately. Nasotr
acheal intubation can be considered, but not before restoring minute volume with
less invasive means. Further assessment will determine the need for a nebulized
beta-2 agonist.
The correct answer is B;
====3/115 Airway
Which of the following statements regarding end-tidal CO2 (ETCO2) monitoring fol
lowing endotracheal intubation is correct?
A: The ETCO2 detector is not affected by endotracheally administered medicat
ions
B: ETCO2 monitoring is most reliable in patients with poor pulmonary perfusi
on
C: Quantitative waveform capnography is extremely accurate and should be use
d
D: Colorimetric ETCO2 detectors provide objective data and are the preferred
device
Reason:
According to the 2010 guidelines for CPR and emergency cardiac care (ECC), quant
itative waveform capnography should be used to confirm initial ET tube placement
and to monitor ongoing ET tube placement. Waveform capnography is extremely acc
urate and provides real-time objective data via an LED reading and a capnographi
c waveform. Because poor pulmonary perfusion results in decreased CO2 eliminatio
n, the ETCO2 detector is less reliable in such cases. Furthermore, elimination a
nd detection of CO2 following endotracheally-administered drugsmost notably epine
phrinecan be drastically reduced. Colorimetric ETCO2 detectors provide qualitativ
e data; they simply detect the presence of carbon dioxide and are less reliable
than waveform capnography, which provides quantitative data.
Correct Answer is C
====4/115 Airway
A 60-year-old woman presents with difficulty breathing. She is conscious and ale
rt, but anxious, and tells you that she was suddenly awakened with the feeling t
hat she was suffocating. She has dried blood on her lips and cyanosis around her
mouth. Her heart rate is 120 beats/min and her oxygen saturation is 89%. Your i
nitial action should be to:
A: administer a bronchodilator.
B: apply supplemental oxygen.
C: attach a cardiac monitor.
D: auscultate breath sounds.
Reason:
Although the patient is conscious and able to converse, she is clearly showing s
igns of hypoxemia (anxiety, oxygen saturation of 89%, tachycardia). Therefore, s
he should be given supplemental oxygen as soon as possible. You must be prepared
to assist her ventilations if her level of consciousness deteriorates or her br
eathing becomes inadequate. If there is evidence of pulmonary edema (ie, rales,
rhonchi), you should consider applying continuous positive airway pressure (CPAP
).
Correct Answer is B
==== 5/115 Airway
Which of the following airway devices is MOST appropriate to use in a deeply unr
esponsive intoxicated patient?
A: Endotracheal tube
B: Perilaryngeal airway
C: Laryngeal mask airway
D: Nasopharyngeal airway
Reason:
Deeply unresponsive patients, especially those who are intoxicated, are at high
risk of regurgitation because their stomach is full of alcohol. The endotracheal
tube virtually eliminates the risk of aspiration because it isolates the trache
a. The laryngeal mask airway (LMA) is a useful alternative airway device that ha
s been shown to provide adequate ventilation; however, it does not eliminate the
risk of aspiration. Perilaryngeal airways, such as the CobraPLA, are also usefu
l alternative airway devices and provide adequate ventilation; however, like the
LMA, they do not eliminate the risk of aspiration. Obviously, a nasopharyngeal
airway will not protect the airway from aspiration; like the oropharyngeal airwa
y, it keeps the tongue away from the posterior pharynx.
Correct Answer is A
==== 6/115 Airway
An unresponsive trauma patient has sonorous respirations and blood draining from
the corner of his mouth. What should be your FIRST action?
A: Suction the oropharynx
B: Ventilate with a bag-mask device
C: Perform a jaw-thrust maneuver
D: Insert an oropharyngeal airway
Reason:
When caring for any unresponsive patient, you must first open their airway. In t
he trauma patient, this involves using the jaw-thrust maneuver. The head tilt-ch
in lift maneuver should be used if the jaw-thrust does not adequately open the a
irway. After opening the patient's airway, ensure that it is clear of any foreig
n bodies or secretions and suction the oropharynx as necessary. After the airway
has been manually opened and cleared of any foreign bodies or secretions, a sim
ple airway adjunct should be inserted to help maintain airway patency. After sec
uring a patent airway, administer oxygen or assist ventilations, as dictated by
the patient's ventilation and oxygenation status.
Correct Answer is C
==== 7/115 Airway
You are ventilating a severely dehydrated apneic 70-year-old male with a history
of end-stage emphysema. In order to minimize the risk of lowering his cardiac o
utput and blood pressure, you should:
A: use a device that provides positive-end expiratory pressure.
B: use a manually-triggered ventilation device.
C: adjust the ventilation rate to allow complete exhalation.
D: hyperventilate the patient at 20 to 24 breaths/min.
Reason:
In patients with severe COPD (ie, end-stage emphysema) and increased resistance
to exhalation, you should attempt to prevent air trapping as this may cause inad
vertent generation of intrinsic positive end-expiratory pressure (also called aut
o-PEEP). In hypovolemic patientsas with your severely dehydrated patientauto-PEEP m
ay significantly reduce cardiac output and blood pressure. Adjusting the ventila
tion rate to approximately 6 to 8 breaths/minwhich will allow for complete exhala
tioncan prevent this. Manually-triggered ventilation devices (eg, demand valve) s
hould not be used in any patient with pulmonary air trapping; use of such device
s may result in widespread alveolar rupture and/or a pneumothorax.
Correct Answer is C
==== 8/115 Airway
Treatment for a patient with an acute asthma attack should focus on which of the
following goals?
A: Determination and correction of the cause
B: Prompt transport to the nearest hospital
C: Rehydration and oxygenation
D: Relief of the bronchospasm and improved ventilation
Reason:
Patients with asthma have two problems: bronchoconstriction and hypoxia; therefo
re, the goals of management are to relieve the bronchospasm and improve ventilat
ion and oxygenation. Dehydration tends to occur in patients with asthma because
of mucous plug formation and drying of the lower airway (children especially); t
herefore, fluid hydration may be required.
Answer is D
==== 9/115 Airway
An adult patient without respiratory distress, who is breathing at a rate of 14
times per minute and has adequate tidal volume, will MOST likely:
A: have adventitious breath sounds upon auscultation.
B: eliminate adequate carbon dioxide from the body.
C: have an arterial PO2 of 60 and 80 torr (mm Hg).
D: maintain a constant oxygen saturation of 100%.
Reason:
Adequate breathing in the adult is characterized by, among other signs, a respir
atory rate between 12 and 20 breaths/min, adequate tidal volume (depth of breath
ing), and a regular pattern of inhalation and exhalation. An adequately breathin
g patient will be able to eliminate adequate amounts of carbon dioxide from the
body, maintain an oxygen saturation (SpO2) of greater than 95%, and will likely
have a normal arterial PO2 of 80 to 100 torr (mm Hg). Adventitious breath sounds
are abnormal breath sounds (ie, wheezing, rales, rhonchi), and may indicate ina
dequate ventilation.
Answer is B
==== 10/115 Airway **REPEAT!***
After determining that your unresponsive patient has a severe (complete) upper a
irway obstruction caused by a foreign body, you should:
A: administer subdiaphragmatic thrusts.
B: reattempt to ventilate the patient.
C: reposition the airway.
D: perform chest compressions.
Reason:
Once you have determined that your unresponsive patient has a severe (complete)
foreign body airway obstruction (eg, you have attempted to ventilate, reposition
ed the airway, and still cannot ventilate), you should perform 30 chest compress
ions, open the airway and visualize the mouth, and remove the foreign body only
if you can see it. Subdiaphragmatic thrusts (Heimlich maneuver) are indicated fo
r responsive adults and children with a severe foreign body airway obstruction.
Answer is D
==== 11/115 Airway
An older woman presents with respiratory distress. She is conscious and alert an
d is able to answer your questions with slight difficulty. Her respirations are
24 breaths/min and labored and her oxygen saturation is 89%. Further assessment
reveals slight cyanosis around her mouth. You should:
A: sedate the patient and perform intubation.
B: apply a nasal cannula at 4 to 6 L/min.
C: assist her breathing with a bag-mask device.
D: apply a nonrebreathing mask at 15 L/min.
Reason:
The fact that the patient is conscious and alert and able to answer your questio
ns, albeit with slight difficulty, indicates that her airway is patent and her b
reathing is adequate. However, cyanosis and a low oxygen saturation are clinical
indicators of hypoxemia and should be treated with high-flow oxygen. In this pa
tient, it would be appropriate to apply a nonrebreathing mask at 15 L/min and cl
osely monitor the adequacy of her breathing. If her level of conscious deteriora
tes and she shows other signs of inadequate breathing (ie, fast or slow breathin
g rate, shallow depth of breathing [reduced tidal volume]), you should assist he
r ventilations with a bag-mask device. This patient is not a candidate for rapid
-sequence intubation because she is able to maintain her own airway.
Answer is D
==== 12/115 Airway
A hypoxemic patient:
A: has a deficiency of oxygen at the cellular level.
B: will likely develop aerobic metabolism.
C: should be intubated and hyperventilated with oxygen.
D: has a decreased oxygen level in the arterial blood.
Reason:
Hypoxemia is a condition in which the oxygen content of arterial blood is decrea
sed. Hypoxia is a dangerous condition in which there is insufficient oxygen at t
he cellular level. Left untreated, hypoxemia will lead to hypoxia and anaerobic
metabolismmetabolism in the absence of oxygenthe byproduct of which is lactic acid
. Aerobic metabolism is a state of normal metabolism in which oxygen is present;
the byproducts of aerobic metabolism are carbon dioxide and water. Intubation i
s indicated for unresponsive patients who are unable to protect their own airway
. Hyperventilation should be avoided because it increases intrathoracic pressure
significantly, which may impede venous return to the heart (preload) and cause
hypotension.
Answer is D
==== 13/115 Airway
When providing positive-pressure ventilation (PPV) to a patient, it is important
to remember that:
A: PPV can impair venous return and cause hypotension.
B: PPV draws blood back to the heart and improves cardiac output.
C: bradycardia is commonly associated with any form of PPV.
D: uncontrolled PPV often causes significant hypertension.
Reason:
During negative-pressure ventilation, the act of normal breathing, air is drawn
into the lungs when intrathoracic decreases; this draws blood back to the heart
and maintains cardiac output. Positive-pressure ventilation (PPV), the act of fo
rcing air into the lungs, increases intrathoracic pressure and can impair venous
return to the heart (preload), resulting in decreased cardiac output and hypote
nsion. For this reason, it is important to ventilate the patient with just enoug
h volume to produce visible chest rise and to avoid hyperventilating the patient
. Bradycardia and hypertension are not commonly associated with PPV.
Answer is A
====14/115 Airway
A 30-year-old man overdosed on codeine and has respirations of 6 breaths/min and
shallow. Which of the following conditions will he develop initially?
A: Metabolic acidosis
B: Respiratory acidosis
C: Metabolic alkalosis
D: Respiratory alkalosis
Reason:
As respirations decrease in rate and depth (tidal volume), carbon dioxide is ret
ained by the body. This leads to an initial state of respiratory acidosis. If le
ft untreated, metabolic acidosis will result as the cells of the body begin prod
ucing lactic acid secondary to anaerobic metabolism. Remember that the initial t
reatment for acidosis, regardless of the underlying cause, is adequate ventilati
on.
Answer is B
==== 15/115 Airway
During an intubation attempt, you are having difficulty viewing the patient's vo
cal cords. Which of the following actions would MOST likely help?
A: Place the patient's head in a hyperextended position
B: Ask your partner to manipulate the external larynx
C: Flex the patient's neck and pull up on the chin
D: Switch to a laryngoscope handle that is larger
Reason:
If you experience difficulty when attempting to view a patient's vocal cords dur
ing intubation, you can ask your partner to perform the BURP maneuver (external
laryngeal manipulation), which involves applying Backward, Upward, and Rightward
Pressure to the larynx; this technique often improves your laryngoscopic view o
f the vocal cords. You can also insert a gum elastic bougie through the vocal co
rds, stroke the wall of the trachea with the bougie (you should feel the "bumps"
of the tracheal wall), and then feed the ET tube over the bougie and into the t
rachea. The patient's head should be in the sniffing position during intubation,
not hyperextended or flexed. Regardless of the size laryngoscope handle that yo
u use, the technique of laryngoscopy is the same; switching to a larger (or smal
ler) handle will likely not help improve your laryngoscopic view of the vocal co
rds.
Answer is B
==== 16/115 Airway
An 16-year-old male presents with acute respiratory distress. His mother tells y
ou that he recently lost his job. He is conscious and alert, but obviously anxio
us. He has a respiratory rate of 40 breaths/min and an oxygen saturation of 98%
on room air. Further assessment reveals carpopedal spasms to his hands. Initial
management for this patient should include:
A: providing coaching to slow the patients breathing.
B: applying an oxygen mask with the flow rate set at 2 L/min.
C: applying high-flow oxygen via nonrebreathing mask.
D: administering a sedative drug to help calm the patient.
Reason:
On the basis of the patient history and physical findings, this case is consiste
nt with an anxiety attack and hyperventilation syndrome. These patients initiall
y need emotional support and respiratory coaching. If the patients respirations d
o not slow down after a reasonable period of coaching, you should consider the p
resence of another underlying cause, such as hypoxemia, and administer high-flow
oxygen. Rebreathing into a paper bag or any other action that causes the patien
t to rebreathe his or her own carbon dioxide (ie, face mask with low-flow oxygen
) is potentially dangerous and should be avoided. The causes of hyperventilation
are many; acute anxiety is but one of them. If you are able to successfully con
trol the patient's breathing, yet he is still experiencing anxiety, it would not
be unreasonable to administer a sedative drug, such as Valium, provided that hi
s vital signs are stable and his breathing remains adequate.
Answer is A
==== 17/115 Airway
After intubating a cardiac arrest patient, you observe a capnography reading tha
t is steadily decreasing. During each ventilation, you see obvious bilateral che
st rise and ventilatory compliance is good. You should:
A: give 1 mEq/kg of sodium bicarbonate.
B: perform tracheobronchial suctioning at once.
C: hyperventilate the patient at 24 breaths/min.
D: reevaluate the effectiveness of your CPR.
Reason:
Quantitative waveform capnography should be used to assess initial advanced airw
ay placement, as well as to monitor ongoing placement. In addition, capnography
can be used as a measure of perfusion. If you note a steady decline in the PETCO
2, especially if it falls below 10 mm Hg, your initial action should be to reeva
luate the effectiveness of your CPR and attempt to improve the quality of chest
compressions. In the context of cardiac arrest, a falling PETCO2 indicates decre
ased carbon dioxide return to the lungs, which may be the result of inadequate C
PR. Hyperventilation should be avoided as this has been shown to impair venous r
eturn (and cardiac output) secondary to increased intrathoracic pressure. There
is no indication in this scenario that the ET tube needs to be suctioned; ventil
ations are producing obvious chest rise and ventilation compliance is good. Rout
ine administration of sodium bicarbonate during cardiac arrest should be avoided
; ensure adequate ventilation and chest compressions first!
Answer is D
==== 18/115 Airway
Which of the following clinical signs is MOST indicative of adequate breathing?
A: Tachypnea and hypopnea
B: Reduced tidal volume
C: Pink oral mucous membranes
D: Unilateral chest wall movement
Reason:
Signs of adequate breathing include bilateral chest wall movement, adequate dept
h (tidal volume), pink skin (including mucous membranes), and an adequate rate.
Tachypnea (rapid breathing) and hypopnea (shallow breathing [reduced tidal volum
e]) will result in a decreased minute volume
Answer is C
==== 19/115 Airway
Common signs of a laryngeal fracture include all of the following, ***EXCEPT***
A: dysphagia.
B: stridor.
C: hematemesis.
D: hemoptysis.
Reason:
Laryngeal fractures are most often caused by blunt trauma to the anterior neck,
usually following a motor-vehicle crash. You should suspect a laryngeal fracture
if a trauma patient presents with painful or difficult swallowing (dysphagia),
coughing up blood (hemoptysis), hoarseness, or difficulty speaking (dysphonia).
As soft tissues of the upper airway swell, air movement becomes restricted; this
results in stridora high-pitched sound heard during inhalation. Vomiting blood (
hematemesis) is not a common clinical sign in patients with a laryngeal fracture
, although it may be observed if the patient swallows large amounts of blood.
Answer is C
==== 20/115 Airway
You are transporting a patient with respiratory distress when he suddenly become
s unresponsive and does not appear to be breathing. The paramedic should:
A: insert a supraglottic airway device.
B: prepare for immediate endotracheal intubation.
C: assess for a carotid pulse for up to 10 seconds.
D: open the airway and give 2 rescue breaths.
Reason:
If an adult patient becomes unresponsive and apneic (as evidenced by an absence
of obvious chest movement) in your presence or is found to be unresponsive and a
pneic, you should assess for a carotid pulse for 5 to 10 seconds. If the patient
has a pulse, open the airway and provide rescue breathing at a rate of 10 to 12
breaths/min (one breath every 5 to 6 seconds). If the patient does not have a p
ulse, immediately perform 30 chest compressions, then open the airway and provid
e two rescue breaths. The technique of look, listen, and feel is no longer recom
mended because it delays the initiation of chest compressions in patients who ar
e in cardiac arrest. Unless airway patency and adequate ventilation cannot be ma
intaned with less invasive means (ie, oral/nasal airway and bag-mask ventilation
), advanced airway management should not be an immediate priority during the ini
tial management of a patient in respiratory or cardiac arrest.
Answer is C
==== 21/115 Airway
Which of the following techniques or devices will provide the highest tidal volu
me to a patient?
A: One-person bag-mask device technique
B: Continuous positive airway pressure (CPAP)
C: Nonrebreathing mask at 15 L/min
D: Pocket face mask with oxygen attached
Reason:
Of the choices listed, the pocket face mask would provide the highest tidal volu
me. This is because the rescuer is breathing air from his/her own lungs into the
patients lungs and both of his/her hands are freed to maintain an effective mask
-to-face seal. The bag-mask device can provide adequate tidal volume; however, g
reater tidal volumes are achieved when two rescuers are ventilating the patient.
In the two-rescuer bag-mask technique, one rescuer maintains a mask seal while
the other squeezes the bag. The nonrebreathing mask is a device that delivers ox
ygen passively; it does not deliver positive-pressure. Continuous positive airwa
y pressure (CPAP) relies upon adequate tidal volume to be effective; the patient
receives maximum benefit from CPAP during the exhalation phase, in which positi
ve-pressure is directed to the lower airways, which forces fluid from the alveol
i and opens the bronchioles.
Answer is D
=== 22/115 Airway
Which of the following is MOST consistent with inadequate breathing in an adult?
A: Expiratory wheezing and pink, moist skin
B: Audible rhonchi and flushed, warm skin
C: Eupneic respirations and warm, dry skin
D: 14 breaths/min with reduced tidal volume
Reason:
Respirations of 14 breaths/min fall within the normal range for an adult. Howeve
r, if the patient's breathing is shallow (reduced tidal volume), pulmonary respi
ration will become inadequate and minute volume will decrease. When evaluating v
entilation adequacy, it is important to note the rate, depth, and regularity of
breathing; do not rely solely on one parameter. The term eupnea is defined as a
normal rate, depth, and regularity of breathing. Other signs of inadequate breat
hing include an irregular pattern of inhalation and exhalation; cool, pale, clam
my skin; cyanosis (a later sign); and a decreased level of consciousness.
Answer is D
==== 23/115 Airway
When attempting visualization of the vocal cords with a curved laryngoscope blad
e, you should:
A: place the tip of the blade directly under the epiglottis and then lift wi
th the long axis of the handle.
B: insert the blade in the midline of the mouth and then gently lift when th
e tip of the blade is in the vallecula.
C: insert the blade into the left side of the mouth, displace the tongue to
the right, and then gently lift.
D: place the tip of the blade in the vallecula, and lift the jaw, tongue, an
d blade gently at a 45 angle.
Reason:
The curved blade is designed to fit into the vallecular space, indirectly lift t
he epiglottis, and expose the vocal cords. The straight blade is placed under th
e epiglottis, directly lifting it to expose the vocal cords. Regardless of which
blade you use, the proper technique of laryngoscopy involves inserting the blad
e in the right side of the mouth, displacing the tongue to the left, and gently
liftingnot pryingat a 45 angle.
Answer is D
==== 24/115 Airway
After inserting an advanced airway device in an adult patient who is in cardiac
arrest, you should:
A: instruct the compressor to stop compressions while you deliver ventilatio
ns.
B: deliver each breath over a period of 2 seconds at a rate of 20 breaths/mi
n.
C: instruct the compressor to continue compressions as you hyperventilate th
e patient.
D: deliver each breath over a period of 1 second at a rate of 8 to 10 breath
s/min.
Reason:
When ventilating a cardiac arrest patient after an advanced airway device has be
en inserted (eg, ET tube, multilumen airway, supraglottic airway), do not synchr
onize compressions with ventilations. Perform continuous chest compressions at a
rate of at least 100/min and deliver ventilations at a rate of 8 to 10 breaths/
min (one breath every 6 to 8 seconds). Deliver each ventilation over a period of
1 second while observing for visible chest rise. Hyperventilating the patient s
hould be avoided because it increases intrathoracic pressure, which can impair v
enous return to the heart and reduce chest compression effectiveness.
Answer is D
==== 25/115 Airway
In which of the following situations would an endotracheal (ET) tube be of LEAST
benefit to your patient?
A: Prolonged ventilation
B: Cardiac arrest
C: Medication administration
D: Unresponsiveness
Reason:
Research and evidence indicates that the endotracheal tube is not a reliable rou
te for medication administration as was once thought. In fact, the onset of acti
on of certain lipid-soluble drugs that can be given endotracheally (ie, lidocain
e, epinephrine, atropine, naloxone) is unpredictable at best. Furthermore, the o
ptimum endotracheal dose of those same drugs is unknown. Endotracheal intubation
is indicated and is beneficial for patients in cardiac arrest, those who are un
responsive and unable to protect their own airway, or when prolonged positive-pr
essure ventilation is required.
Answer is C
==== 26/115 Airway
Which of the following would MOST likely occur if an adult patient is breathing
at a rate of 36 breaths/min and shallow?
A: Minute volume would remain unchanged.
B: Tidal volume would increase.
C: More air would linger in the anatomic dead space.
D: Tidal volume and minute volume would increase.
Reason:
A respiratory rate of 36 breaths/min and shallow (reduced tidal volume) would re
sult in lesser amounts of air reaching the alveoli and participating in pulmonar
y gas exchange (pulmonary respiration). As the respiratory rate becomes faster,
they also become more shallow. This would result in more air remaining in the an
atomic dead space (ie, trachea, large bronchi); as a result, minute volume would
decrease.
Answer is C
==== 27/115 Airway
Which of the following breath sounds is also referred to as crackles?
A: Rhonchi
B: Vesicular
C: Rales
D: Wheezing
Reason:
Rales are commonly referred to as crackles. They are fine, moist, thin sounds th
at are often difficult to hear and indicate fluid in the smaller lower airways,
such as what occurs during early pulmonary edema. Rhonchi is much louder than ra
les and produce a rattling sound; it indicates fluid in the larger lower airways
and is often encountered in patients with severe pulmonary edema or bronchitis.
Wheezing is a whistling sound that indicates bronchospasm; wheezes may be heard
during inspiration, expiration, or both, and are most often encountered in pati
ents with reactive airway diseases (ie, asthma, bronchiolitis). Vesicular breath
sounds, which are normal, are characterized by inspiratory sounds that last lon
ger than expiratory sounds; they are normally faint.
Answer is C
==== 28/115 Airway
Assessment of a patient with acute respiratory distress reveals that he is consc
ious and alert, but wheezing on exhalation. In addition to oxygen, management sh
ould include:
A: administration of a beta-blocker, such as metoprolol tartrate.
B: administration of an inhaled beta-2 agonist medication.
C: pharmacologically-assisted intubation.
D: administration of 3 to 5 mg of epinephrine subcutaneously.
Reason:
Selective beta-2 agonists, such as albuterol (Ventolin, Proventil) or metaproter
enol sulfate (Alupent), are typically given via a nebulizer for patients with re
active airway diseases (ie, asthma, bronchiolitis, etc) to promote bronchodilati
on and improve ventilation. Continuous positive airway pressure (CPAP) has also
shown to be effective in treating patients with acute bronchospasm, especially w
hen used in conjunction with an in-line nebulizer and beta-2 agonist. Epinephrin
e 1:1,000, in a dose of 0.3 to 0.5 mg (0.3 to 0.5 mL) subcutaneously, could also
be administered to patients with acute bronchospasm. A patient with bronchospas
m, as evidenced by wheezing, must never be given a beta-blocker as this may exac
erbate their condition and could lead to respiratory failure or arrest. Some pat
ients with severe refractory bronchospasm may require intubation; this is especi
ally true if their level of consciousness deteriorates. However, the patient in
this scenario is conscious and alert.
Answer is B
==== 29/115 Airway
Common clinical signs of respiratory distress include all of the following, EXCE
PT:
A: pursed-lip breathing.
B: a flushed appearance.
C: circumoral cyanosis.
D: intercostal retractions.
Reason:
Flushed (red) skin is not a common clinical sign of respiratory distress. It is
more commonly seen in patients with a fever or in those who have been exposed to
high environmental temperatures. Common signs of respiratory distress include i
ntercostal retractions, accessory muscle use, pursed-lip breathing (especially c
ommon in patients with emphysema), and, as a later sign, cyanosis. Cyanosis arou
nd the mouth is called circumoral (or perioral) cyanosis.
Answer is B
==== 30/115 Airway
An unresponsive man is brought to the emergency department by his wife. Initial
arterial blood gas analysis reveals a pH of 7.1, a PaO2 of 68 mm Hg, and a PaCO2
of 60 mm Hg. These findings are MOST consistent with:
A: metabolic acidosis.
B: respiratory acidosis.
C: respiratory alkalosis.
D: metabolic alkalosis.
Reason:
The patient is experiencing respiratory acidosis. A pH of less than 7.35 indicat
es acidosis; alkalosis of any kind is quickly ruled out by simply noting the low
pH. An elevated (greater than 45 mm Hg) PaC02 indicates carbon dioxide retentio
n, and a low (less than 80 mm Hg) PaO2 indicates hypoxia. This blood gas report
is consistent with a patient who is hypoventilating.
Answer is B
==== 31/115 Airway
Which of the following statements regarding Cheyne-Stokes respirations is correc
t?
A: They are not considered ominous unless grossly exaggerated or in the cont
ext of a traumatic brain injury.
B: Cheyne-Stokes respirations always indicate increased intracranial pressur
e and brainstem dysfunction.
C: It is common to observe Cheyne-Stokes respirations in patients who have o
verdosed on narcotic drugs.
D: They are characterized by an irregular pattern, rate, and depth of respir
ation with intermittent apneic periods.
Reason:
Cheyne-Stokes respirations are more of a high-brain function. Many deep sleepers
or intoxicated people exhibit this respiratory pattern. The depth of breathing
(or volume of snoring) gradually increases, then decreases (crescendo-decrescend
o), followed by an apneic period. Despite their abnormal appearance, Cheyne-Stok
es respirations are not considered ominous unless they are grossly exaggerated o
r occur in the context of a traumatic brain injury. Biots respirations, also call
ed ataxic respirations, are characterized by an irregular pattern, rate, and dep
th of breathing with intermittent apneic periods; they indicate a severe brain i
njury. Overdose of a narcotic (opiate) drug, such as heroin or morphine, would b
e expected to cause slow, shallow breathing secondary to depression of the respi
ratory centers in the brain.
Answer is A
==== 32/115 Airway
While intubating a 44-year-old man in respiratory arrest, you note that his puls
e rate increases during the procedure. What should you do?
A: Discontinue the intubation attempt and hyperventilate the patient for 2 t
o 3 minutes.
B: Complete the intubation attempt and administer adenosine after the tube i
s secured.
C: Recognize this as a normal response during intubation and monitor the pul
se rate.
D: Continue your intubation attempt as your partner performs a carotid sinus
massage.
Reason:
Endotracheal intubation typically causes stimulation of the sympathetic nervous
system; therefore, it is common to see increases in both the pulse rate and bloo
d pressure during the procedure. This is usually handled well in most patients,
provided that they do not have concomitant head injury with increased intracrani
al pressure (ICP). In cases such as this, lidocaine can be considered, which has
been shown, albeit through an unknown mechanism, to transiently blunt a sudden
rise in ICP during intubation. Routine hyperventilation should be avoided as it
has been shown to increase intrathoracic pressure and impair venous return to th
e heart. Ventilate the apneic adult at a rate of 10 to 12 breaths/min (one breat
h every 5 to 6 seconds).
Answer is C
==== 33/115 Airway
Which of the following would occur as the result of central neurogenic hypervent
ilation?
A: Hypercarbia and respiratory acidosis
B: Hypocarbia and respiratory alkalosis
C: Hypercarbia and metabolic acidosis
D: Hypocarbia and metabolic alkalosis
Reason:
Central neurogenic hyperventilation is characterized by deep, rapid respirations
(hyperpnea and tachypnea) and is caused by increased intracranial pressure. Bec
ause the respirations are rapid and deep, they drive carbon dioxide levels down
(hypocarbia) and pH levels up, resulting in respiratory alkalosis.
Answer is B
==== 34/115 Airway
A 40-year-old woman who was recently discharged from the hospital reports a sudd
en onset of difficulty breathing and sharp chest pain that increases with breath
ing. Her skin remains cyanotic and her oxygen saturation remains low, despite hi
gh-flow oxygen. Which of the following is this patient MOST likely experiencing?
A: Spontaneous pneumothorax
B: Acute pulmonary embolism
C: Acute pulmonary artery rupture
D: Acute bacterial pneumonia
Reason:
This is a rather classic presentation of an acute pulmonary embolism, which is c
haracterized by a sudden onset of difficulty breathing and sharp (pleuritic) che
st pain. Persistent cyanosis and an oxygen saturation level that remains low des
pite high-flow oxygen are also common findings, especially in patients with a la
rge pulmonary embolus. Patients who have been recently hospitalized or otherwise
immobile for a prolonged period of time are at risk of developing a pulmonary e
mbolus. This is because blood stagnates in the lower extremities, resulting in t
he eventual formation of a thrombus (clot), which breaks free, travels to the lu
ngs, and lodges in a pulmonary artery.
Answer is B
==== 35/115 AirwayA 21-year-old man is unresponsive and has shallow, gurgling re
spirations. After manually opening his airway, the paramedic should:
A: attach a pulse oximeter.
B: prepare for intubation.
C: begin assisting ventilations.
D: suction the oropharynx.
Reason:
To effectively manage a patients airway, you must ensure that it is open and clea
r of foreign bodies, blood, or other secretions. The presence of gurgling is an
indication that the airway contains secretions; therefore, the patient's orophar
ynx must be suctioned prior to any further interventions. Aspiration increases m
ortality significantly!
Answer is D
==== 36/115 Airway
What is the approximate minute alveolar volume of a patient who breathes in 550
mL of air at a rate of 14 times per minute?
A: 5.4 L
B: 7.7 L
C: 6.2 L
D: 4.8 L
Reason:
Tidal volume is the amount of air that moves into or out of the respiratory trac
t per breath. Alveolar volume is the amount of air that actually reaches the lun
gs per breath. Minute alveolar volume, also called minute alveolar ventilation,
is calculated by multiplying the patients tidal volume, minus dead space volume,
and the respiratory rate. Approximately 30% of a persons tidal volume (or about 1
mL per pound of body weight) lingers in the anatomic dead space (eg, trachea, l
arger bronchi) and does not reach the lungs to participate in gas exchange. Ther
efore, a patient with an alveolar volume of 385 mL (550 165 [30% of 550] = 385)
and a respiratory rate of 14 breaths/min has an approximate minute alveolar volu
me of 5.4 L (385 14 = 5,390 mL [5.4 L]).
Answer is A
==== 37/115 Airway
You are assessing an unresponsive man's respirations and note that he is taking
irregular breaths that vary in volume and rate with periods of apnea. This breat
hing pattern is MOST consistent with:
A: agonal respirations.
B: Cheyne-Stokes respirations.
C: Biots respirations.
D: Kussmauls respirations.
A pattern of irregular breathing that varies in depth and rate with sudden perio
ds of apnea is referred to as Biots respirations, also known as ataxic breathing.
This respiratory pattern is commonly seen in patients with increased intracrani
al pressure, either from closed head trauma or hemorrhagic stroke. Clearly, pati
ents displaying this respiratory pattern need ventilatory assistance. Agonal res
pirations, also called agonal gasps, are slow and irregular and produce minimal
to no tidal volume; they are commonly observed shortly after a patient develops
cardiac arrest. Kussmaul respirations are characterized by a pattern of deep and
rapid breathing and are commonly observed in patients with diabetic ketoacidosi
s. Cheyne-Stokes respirations follow a crescendo-decrescendo pattern in which th
e patient breaths fast, then slow, and has intermittent periods of apnea.
Answer is C
==== 38/115 Airway
Which of the following processes occurs during inhalation?
A: The diaphragm contracts and descends, intrathoracic pressure decreases, a
nd air enters the lungs via negative pressure.
B: The intercostal muscles relax, the ribcage expands, intrathoracic pressur
e increases, and air enters the lungs via positive pressure.
C: The diaphragm contracts and ascends, intrathoracic pressure decreases, an
d air enters the lungs via positive pressure.
D: The intercostal muscles and diaphragm contract, intrathoracic pressure in
creases, and air passively enters the lungs.
Reason:
During inhalationan active processthe diaphragm contracts and descends and the int
ercostal muscles contract. These processes cause an increase in the vertical and
horizontal dimensions of the thoracic cavity. As a result, intrathoracic pressu
re falls and air is drawn into the lungs via negative pressure. Exhalation is a
passive process that occurs when the diaphragm and intercostal muscles relax and
air exits the lungs passively. Positive pressure ventilation is the act of forc
ing air into the lungs and is not a part of normal breathing; it is provided by
the paramedic in the form of artificial ventilation (eg, rescue breathing).
Answer is A
==== 39/115 Airway
You are attempting to ventilate a patient with a bag-mask device and do not see
the chest visibly rise. What should your initial course of action be?
A: Reposition the patients head.
B: Switch to a pocket face mask.
C: Insert a supraglottic airway device.
D: Suction the oropharynx for 15 seconds.
Reason:
The first step that should be taken if the chest does not visibly rise with arti
ficial ventilation is to reposition the head to ensure that the tongue is not bl
ocking the airway. If there are secretions in the mouth, suction the oropharynx
for no more than 15 seconds (10 seconds in children; 5 seconds in infants). If t
here are no secretions in the airway and repositioning the head does not allow f
or adequate artificial ventilation, consider that the patient has a foreign body
airway obstruction. Advanced airway devices (ie, ET tube, multilumen airway, su
praglottic airway) are indicated for patients who are unresponsive, cannot maint
ain their own airway, and require prolonged ventilatory support.
Answer is A
==== 40/115 Airway
You are ventilating an intubated patient and note decreased compliance with each
delivered ventilation. Which of the following conditions would be the LEAST lik
ely cause of this?
A: Beta-2 receptor stimulation
B: Diffuse bronchospasm
C: Tension pneumothorax
D: Surfactant deficiency
Reason:
Decreased ventilatory compliance (increased ventilatory resistance) is the resul
t of any condition that causes a barrier to airflow into the lungs. Such conditi
ons include bronchospasm, tension pneumothorax, upper or lower airway obstructio
n, and surfactant deficiency. Surfactant is a protein substance that lines the a
lveolar walls and decreases alveolar surface tension. Stimulation of beta-2 rece
ptors causes bronchodilation, which would facilitatenot impedeairflow into the lun
gs.
Answer is A
==== 41/115 Airway
Which of the following statements regarding the concentration of gases is correc
t?
A: Atmospheric air contains approximately 25% to 30% oxygen.
B: Nitrogen accounts for approximately 79% of atmospheric air.
C: The partial pressure of alveolar oxygen is about 75 torr.
D: Fifteen percent of the gas within the alveoli is carbon dioxide.
Answer is B
The majority of atmospheric gas is composed of nitrogenapproximately 79%. The par
tial pressure of oxygen in the alveoli is approximately 104 torr (mm Hg). Atmosp
heric (room) air contains 20.8% oxygen. The percentage of alveolar carbon dioxid
e is approximately 5%.
Answer is B
==== 42/115 Airway
Tidal volume is defined as the:
A: volume of air moved in and out of the lungs each minute.
B: volume of air moved in and out of the lungs per breath.
C: residual volume of air in the lungs at the end of exhalation.
D: maximum volume of air that the lungs can accommodate.
Reason:
Tidal volume is defined as the volume of air (in mL) that is moved in and out of
the respiratory tract in a single breath. The volume of air moved in and out of
the respiratory tract each minute is called minute volume, and is measured in l
iters. The average tidal volume of an adult male is approximately 500 mL. Of thi
s amount, approximately 150 mL (1 mL per pound of body weight) remains in the an
atomic dead space (trachea, large bronchi) and never reaches the alveolar level.
Therefore, of the 500 mL of air that a 150-pound patient inhales, 350 mL actual
ly reaches the alveoli (alveolar volume) and participates in pulmonary respirati
on; the remaining 150 mL lingers in the anatomic dead space until it is exhaled.
Answer is B
==== 43/115 Airway
Initial management of an unresponsive 20-year-old patient with respirations of 1
4 breaths/min and adequate depth should include:
A: inserting an airway adjunct and suctioning the mouth every 30 seconds.
B: inserting an airway adjunct and giving oxygen via nonrebreathing mask.
C: inserting an airway adjunct and providing assisted ventilations.
D: performing immediate intubation to protect the patients airway.
Reason:
Because the patient is unresponsive, an airway adjunct (oral or nasal airway) sh
ould be inserted, which, along with manual airway positioning, will help maintai
n airway patency. The patient in this scenario has adequate ventilation (14 brea
ths/min with adequate depth [tidal volume]); therefore, high-flow oxygen via non
rebreathing mask would be the most appropriate initial approach. It is essential
to continue to monitor the patient for signs of inadequate ventilation, and be
prepared to assist ventilations if needed. Advanced airway management should be
considered if the unresponsive patient requires ventilatory support, especially
if the need for ventilatory support will be prolonged.
Answer is B
==== 44/115 Airway
In which of the following conditions would you MOST likely detect a drop in syst
olic blood pressure during inhalation?
A: Severe asthma
B: Simple pneumothorax
C: Pulmonary edema
D: Chronic bronchitis
Reason:
Pulsus paradoxus, a drop in the systolic blood pressure of 10 mm Hg or more duri
ng inhalation, indicates physical restriction of cardiac movement during inhalat
ion. Pulsus paradoxus can also be characterized by a marked weakening (or even d
isappearance) of the pulse during inhalation. Pulsus paradoxus may be observed i
n patients experiencing a severe asthma attack or an exacerbation of emphysema,
in which case air-trapping in the lungs leads to pulmonary hyperinflation. As th
e hyperinflated lungs put pressure against the heart during inhalation, cardiac
movement can be physically restricted. Pulsus paradoxus may also be observed in
patients with a severe pericardial tamponade.
Answer is A
==== 45/115 Airway
Which of the following statements regarding physiologic dead space is correct?
A: Unlike anatomic dead space, the size of the physiologic dead space does n
ot change.
B: Diffuse alveolar collapse increases the size of the physiologic dead spac
e.
C: COPD patients typically have a marked decrease in physiologic dead space.
D: Physiologic dead space is normally found in the trachea and large bronchi
.
Reason:
Physiologic dead space is any portion of the lower respiratory tract in which ga
s exchange does not occur because of abnormal processes such as alveolar collaps
e (atelectasis), alveolar damage, or fluid in the alveoli. Because COPD causes d
estruction of the inner lining of the alveoli, physiologic dead space increases
in patients with the disease. Approximately 30% of a persons tidal volume remains
in the anatomic dead spacethe trachea and large bronchiand never makes it to the
lungs to participate in pulmonary respiration. Unlike the physiologic dead space
, which increases in size in patients with respiratory diseases, the size of the
anatomic dead space remains relatively constant.
Answer B
==== 46/115 Airway
You would MOST likely encounter bradypnea in a patient who:
A: ingested salicylates.
B: has metabolic alkalosis.
C: took 5 mg of diazepam.
D: is acutely hypoxemic.
Reason:
Metabolic alkalosis (pH > 7.45) often results in periods of bradypnea (abnormall
y slow respirations). This occurs due to compensation by the respiratory buffer
system, which is attempting to retain carbon dioxide and hydrogen ions in order
to lower the pH. Five milligrams of diazepam (Valium) is a therapeutic dose; CNS
depression (ie, bradypnea, bradycardia, hypotension) would be unlikely at this
dose, unless the patient co-ingested another CNS depressant (eg, opiates). When
the bodys chemoreceptors sense increased arterial CO2 levels (hypercarbia) or dec
reased O2 levels (hypoxemia), the respiratory centers in the brain send more mes
sages to the respiratory muscles; as a result, respirations increase (tachypnea)
in order to bring in more oxygen and eliminate more carbon dioxide. A patient w
ho ingested salicylates (ie, acetylsalicylic acid [aspirin, ASA]) would present
with tachypnea because the respiratory buffer system is attempting to eliminate
excess hydrogen ions by increasing the rate and depth of breathing.
Answer is B
==== 47/115 Airway
You have attempted orotracheal intubation on a cardiac arrest patient, but were
unsuccessful after two attempts. When you resume bag-mask ventilations, you are
unable to maintain an adequate mask-to-face seal. What should you do?
A: Insert a multilumen or supraglottic airway device
B: Have your partner attempt to intubate the patient
C: Perform a surgical cricothyrotomy
D: Perform nasotracheal intubation instead
Reason:
If you are unable to successfully perform orotracheal intubation, and you cannot
provide effective ventilations with the bag-mask device, the quickest and most
practical approach would be to insert an alternative airway device, such as a mu
ltilumen airway (eg, Combitube) or a supraglottic airway (eg, LMA, King LT, Cobr
aPLA), and resume ventilations as soon as possible. These devices are easy to in
sert, have been shown to provide better ventilation than the bag-mask device, an
d are much safer than performing a surgical (open) cricothyrotomy. Do not develo
p tunnel-vision and fixate on orotracheal intubation when alternative airway dev
ices are available. Remember, the goal is to ventilate, not intubate. Nasotrache
al intubation is contraindicated in this patient because he is apneic.
Answer is A
==== 48/115 Airway
A patient was bitten by fire ants and is unresponsive. He has severe edema to th
e face and neck and generalized urticaria. Breath sounds are difficult to hear,
and loud inspiratory stridor is noted. Which of the following interventions has
the highest priority?
A: Immediate transport
B: Administration of epinephrine
C: Administration of diphenhydramine
D: Endotracheal intubation
Reason:
The patients upper airway is rapidly swelling as evidenced by the markedly dimini
shed breath sounds and loud inspiratory stridor. If the patient is not promptly
intubated before the airway completely closes, you will have to perform a cricot
hyrotomy or the patient will die. After the airway is secured and the patient is
being adequately oxygenated and ventilated, you should administer epinephrine,
followed by diphendydramine (Benadryl). Transport the patient as soon as possibl
e.
Answer is D
==== 49/115 Airway
Which of the following clinical presentations is MOST indicative of a severe upp
er airway obstruction?
A: Ability to speak in broken sentences
B: Weak cough and cyanosis
C: Anxiety and a forceful cough
D: Flushed skin and tachycardia
Reason:
A severe airway obstruction is characterized by an inability to speak, minimal o
r absent air movement, extreme anxiety, decreased level of consciousness (ie, co
nfusion, lethargy, unresponsive), falling oxygen saturation, and cyanosis. If th
e patient is coughing, it is weak and ineffective.
Answer B
==== 50/115 Airway
A selective beta-2 adrenergic agonist will produce which of the following effect
s?
A: Bronchodilation
B: Increased parasympathetic tone
C: Increased vascular resistance
D: Increased inotropy
Reason:
Selective beta-2 adrenergic agonists, such as albuterol (Proventil, Ventolin) an
d metaproterenol (Alupent), cause bronchial smooth muscle relaxation, resulting
in bronchodilation. Therefore, their use is indicated for patients with reactive
airway diseases and accompanying bronchospasm. Beta-1 agonists, such as epineph
rine, affect the heart, resulting in increased contractility (inotropy), heart r
ate (chronotropy), and electrical conduction velocity (dromotropy). Alpha agonis
ts, such as norepinephrine (Levophed), stimulate receptors in the vasculature, r
esulting in an increase in vascular resistance and arterial blood pressure. Beta
agonists do not stimulate the parasympathetic nervous system.
Answer A
==== 51/115 Airway ***DUPLICATE***
Factors that increase the amount of energy needed for ventilation include all of
the following, EXCEPT:
A: stimulation of beta-2 receptors.
B: surfactant deficiency.
C: decreased pulmonary compliance.
D: increased airway resistance.
Reason:
The amount of energy needed for normal (unassisted) ventilation in an otherwise
healthy adult is only about 3% of the total body expenditure. Stimulation of bet
a-2 receptors would facilitate ventilation by dilating the bronchioles, and woul
d not increase the amount of energy required for ventilation. Anything that impa
irs ventilationloss of pulmonary surfactant (eg, emphysema), increased airway res
istance (eg, bronchospasm), and decreased pulmonary compliance (eg, pulmonary ed
ema, COPD)can cause a significant increase in the amount of energy needed for ven
tilation, perhaps as high as 35%.
Answer is A
==== 52/115 Airway
Which of the following is the MOST reliable immediate indicator that you have su
ccessfully placed an endotracheal (ET) tube into the trachea?
A: Visualization of the ET tube passing in between the vocal cords.
B: The presence of breath sounds that are clear and equal bilaterally.
C: Paper color change in an end-tidal CO2 detector during exhalation.
D: Vapor mist is visible inside the ET tube during exhalation.
Reason:
Of the choices listed, the most reliable immediate indicator that you have succe
ssfully intubated the trachea is to see the tip of the ET tube pass through the
vocal cords with your own eyes. When you remove the laryngoscope blade from the
patients mouth, however, you must ensure continued correct placement of the ET tu
be. Methods and techniques used to accomplish this include quantitative waveform
capnography (preferred), the presence of clear and equal breath sounds bilatera
lly, and an absence of epigastric sounds. The colorimetric ETCO2 detector provid
es qualitative data regarding correct ET tube placement and is fairly reliable,
but not as reliable as waveform capnography. The esophageal detector device (EDD
) is also an alternative option for confirming correct ET tube placement, but is
also less reliable than waveform capnography. The presence of vapor mist in the
ET tube during exhalation should not be relied upon solely; it can occur with b
oth tracheal and esophageal intubation.
Answer is A
==== 53/115 Airway
An unresponsive, apneic patient has massive facial trauma, including a crushed m
andible and severe oral bleeding. Orotracheal intubation has been attempted twic
e without success. The paramedic should proceed with:
A: needle or surgical cricothyrotomy.
B: insertion of a supraglottic device.
C: blind nasotracheal intubation.
D: digital intubation.
Reason:
In a patient with massive maxillofacial trauma and severe oral bleeding, orotrac
heal intubation may be extremely difficult or impossible to perform. A supraglot
tic airway device (ie, King LT, CobraPLA, LMA) would be of limited benefit becau
se it does not eliminate the threat of aspiration. Digital (tactile) intubation
often takes too long to perform and is frequently unsuccessful, and nasotracheal
intubation is contraindicated in apneic patients. In cases where orotracheal in
tubation is not possible and other forms of airway management are not feasible,
a needle or surgical cricothyrotomy should be performed in order to gain control
over the patient's airway.
Answer is A
==== 54/115 Airway
When arterial oxygen levels in the body fall, chemoreceptors in the brain send m
essages to the diaphragm via the:
A: vagus nerve.
B: phrenic nerve.
C: medulla.
D: brainstem.
Reason:
When low oxygen levels are detected by the chemoreceptors in the blood, messages
are sent to the diaphragm via the phrenic nerve, which originates in between th
e third and fifth cervical vertebrae. Injury to the cervical spine in this area
can sever the phrenic nerve and result in respiratory paralysis. The chemorecept
ors in the brain are located within the medulla oblongata, a part of the brainst
em.
Answer is B
==== 55/115 Airway
You have intubated a 33-year-old woman in cardiac arrest secondary to trauma. Wh
ile auscultating her lungs, you note that breath sounds are absent over the righ
t hemithorax. This clinical finding is MOST suggestive of:
A: intubation of the hypopharyngeal area.
B: blood or air in the right hemithorax.
C: inadvertent intubation of the esophagus.
D: left mainstem bronchus intubation.
Reason:
When breath sounds are unilaterally diminished or absent, especially following t
rauma, intrathoracic injury (ie, tension pneumothorax, hemothorax, hemopneumotho
rax) should be suspected. Anatomically, it is nearly impossible to insert the en
dotracheal tube too far into the left mainstem bronchus because it takes a more
acute angle than the right mainstem bronchus. Inadvertent intubation of the esop
hagus or hypopharyngeal area would result in absent breath sounds bilaterally.
Answer is B
==== 56/115 Airway
A 60-year-old man reports dyspnea. While auscultating his chest, you hear fine,
moist, thin sounds in all lung fields. What is this MOST suggestive of?
A: Mucous plugs in the alveoli
B: Fluid in the large lower airways
C: Mild to moderate bronchospasm
D: Fluid in the small lower airways
Reason:
Crackles (formerly called rales), which are fine, moist, thin sounds that are di
fficult to auscultate, represent fluid in the small lower airways and are indica
tive of early pulmonary edema (ie, congestive heart failure). Rhonchi are loud r
attling sounds that can often be heard without a stethoscope and are indicative
of fluid in the larger airways (ie, severe pulmonary edema). Bronchospasm typica
lly presents with wheezing, not crackles or rhonchi.
Answer is D
==== 57/115 Airway
When performing tracheobronchial suctioning on an adult, it is important to:
A: monitor the patients oxygen saturation and cardiac rhythm.
B: coat the tip of the catheter with a petroleum-based lubricant.
C: hyperventilate the patient before inserting the suction catheter.
D: premedicate the patient with atropine to prevent bradycardia.
Reason:
Tracheobronchial (endotracheal) suctioning not only removes secretions from the
ET tube, but also can remove oxygen from the body. Therefore, it is important to
monitor the patients oxygen saturation and cardiac rhythm during the procedure.
A sudden increase in heart rate and/or decrease in oxygen saturation indicates h
ypoxemia and the need to abort the suction attempt and resume ventilations. If y
our patient is adequately preoxygenated before the suction attempt, the incidenc
e of hypoxemia can be greatly reduced. Do not hyperventilate the patient, howeve
r, as this may impair venous return to the heart and cause hypotension. The tip
of the flexible suction catheter should be coated with a water-soluble lubricant
; a petroleum-based substance would likely obstruct the ET tube. Atropine is not
indicated prior to tracheobronchial suctioning; hypoxemia in the adult would li
kely manifest with tachycardia, not bradycardia.
Answer is A
==== 58/115 Airway
A 49-year-old man presents with acute shortness of breath. He is conscious, but
confused, and is gasping for air. The pulse oximeter reads 79% on room air. Init
ial management should consist of:
A: some form of ventilatory assistance.
B: sedation and endotracheal intubation.
C: insertion of an oropharyngeal airway.
D: 100% oxygen via nonrebreathing mask.
Reason:
Confusion (indicates decreased cerebral perfusion), gasping for air (air hunger)
, and an oxygen saturation of 79% are clear signs that this patient has inadequa
te ventilation; therefore, he will require some form of ventilatory assistance,
such as a bag-mask device with supplemental oxygen. The fact that he is awake ne
gates the use of an oral airway. Should the patient deteriorate further, endotra
cheal intubation should be considered.
Answer is A
==== 59/115 Airway
After placing an endotracheal tube in a cardiac arrest patient, large amounts of
vomitus immediately begin flowing out of the tube. You should:
A: immediately remove the ET tube, resume bag-mask ventilations, and reattem
pt intubation after 2 minutes.
B: leave the ET tube in place, fold it to the side so the vomitus can drain,
and resume bag-mask ventilations.
C: apply posterior pressure to the cricoid cartilage to compress the esophag
us and stop the flow of vomitus.
D: withdraw the ET tube just until the flow of vomitus stops and inflate the
distal cuff with 5 to 10 mL of air.
Reason:
If inadvertant esophageal intubation occurs and vomitus begins flowing out of th
e ET tube, you should leave the ET tube in place, inflate the distal cuff with m
ore air than normal (perhaps as much as 20 to 30 mL), fold the ET tube to the si
de to allow the vomitus to drain, and resume bag mask ventilations. If you remov
e the ET tube when the patient is regurgitating, then you have virtually assured
aspiration. With proper technique, you can maintain an adequate mask seal over
the folded ET tube in order to ventilate the patient with the bag-mask device. A
pplying posterior cricoid pressure may compress the esophagus, but will NOT comp
ress the ET tube that is in the esophagus. Consider attaching suction to the end
of the ET tube to decompress the stomach.
Answer is B
==== 59/115 Airway
The normal partial pressure of oxygen in arterial blood is:
A: 60 to 80 mm Hg.
B: 100 to 120 mm Hg.
C: 35 to 45 mm Hg.
D: 80 to 100 mm Hg.
Reason:
Arterial blood should have a high partial pressure of oxygen because it has been
reoxygenated in the lungs. The normal partial pressure of oxygen in arterial bl
ood (PaO2) is 80 to 100 mm Hg. An arterial PaO2 of less than 80 mm Hg indicates
tissue hypoxia. The normal partial pressure of carbon dioxide in arterial blood
(PaCO2) is 35 to 45 mm Hg.
Answer is D
==== 61/115 Airway
A 50-year-old man with a self-inflicted gunshot wound to the face is apneic. He
has multiple fractures of the mandible, massive soft tissue damage, and severe o
ropharyngeal bleeding. Which of the following methods of airway control will be
MOST effective for this patient?
A: Orotracheal intubation
B: Surgical cricothyrotomy
C: Oral airway and bag-mask ventilation
D: Nasotracheal intubation
Reason:
Managing the airway of a patient with massive maxillofacial trauma can present a
challenge to even the most experienced paramedic. In this case, orotracheal int
ubation would be extremely difficult because of the blood in the patient's mouth
and his mandibular fractures (orotracheal intubation relies on a stable mandibl
e). Nasotracheal intubation is contraindicated because the patient is apneic. Ba
g-mask ventilations will likely be ineffective because maintaining a mask seal w
ill be extremely difficult. Furthermore, bag-mask ventilation in a patient with
active oropharyngeal bleeding will force blood into the lungs, causing the patie
nt to aspirate. Although considered as a last resort, this patient is a candidat
e for a surgical (open) cricothyrotomy.
Answer is B
==== 62/115 Airway
While transporting a man with severe respiratory distress, he pulls the oxygen m
ask from his face and frantically attempts to get off of the stretcher. You shou
ld:
A: hold the mask to his face and provide reassurance.
B: administer 5 mg of diazepam to calm the patient.
C: place the patient supine and assist his ventilations.
D: apply a nasal cannula and try to calm the patient. Reason:
The patient is obviously experiencing significant respiratory distress and is hy
poxic, as evidenced by his extreme restlessness. In cases such as this, your mos
t appropriate action is to attempt to calm the patient and offer oxygen with a l
ess oppressive device, such as a nasal cannula, and closely monitor his breathin
g and level of consciousness. Holding the mask to the patient's face will arguab
ly make him more anxious and increase his body's demand for oxygen. If his level
of consciousness deteriorates, you should begin assisting his ventilations. CNS
depressants, such as diazepam (Valium), should not be given to hypoxic patients
with respiratory distress as it may cause respiratory arrest.
Answer is B
==== 63/115 Airway
A responsive 20-year-old male with a suspected foreign body airway obstruction i
s anxious, coughing forcefully, and is able to speak with difficulty. You should
:
A: perform subdiaphragmatic thrusts with the patient in a standing position.
B: closely monitor the patient's condition and encourage him to keep coughin
g.
C: insert a nasopharyngeal airway to maintain airway patency.
D: perform laryngoscopy and remove the obstruction with Magill forceps.
Reason:
Patients with a mild airway obstruction (eg, strong cough, adequate mental statu
s, normal oxygen saturation, ability to speak with difficulty) are able to move
enough air to maintain adequate oxygenation. Leave these patients alone! Closely
monitor the patient, encourage him or her to continue to cough, offer supplemen
tal oxygen, and transport to the hospital. Efforts to treat a mild airway obstru
ction with subdiaphragmatic (abdominal) thrusts may convert a mild airway obstru
ction to a severe one. Obviously, laryngoscopy is contraindicated in any conscio
us patient.
Answer is B
==== 64/115 Airway
Normal breathing in a resting adult male:
A: should be marked by only subtle changes in rate or regularity.
B: is characterized by a short inspiratory time and prolonged expiratory tim
e.
C: typically causes a weakening of the pulse during peak inspiration.
D: generally produces a tidal volume of about 8 to 10 mL/kg.
Reason:
Breathing should be effortless, with inspiratory and expiratory times being roug
hly equal. A brief (or normal) inspiratory time followed by a prolonged expirato
ry time is a sign of inadequate ventilation. The healthy adult male has a tidal
volume of 5 to 7 mL/kg (about 500 mL). During normal breathing, subtle changes i
n rate and/or regularity are normal; grossly obvious changes, however, are not.
A weakening of the pulse during peak inspirationcalled pulsus paradoxusis not norm
al, and is often associated with conditions such as severe asthma, COPD, and per
icardial tamponade.
Answer is A
==== 65/115 Airway
While attempting to ventilate an apneic patient with a stoma, you note minimal r
ise of the chest and can hear air escaping through the upper airway. You should:
A: insert a nasopharyngeal airway and suction the stoma for no more than 5 s
econds.
B: perform an emergency tracheotomy just inferior to the site of the stoma.
C: pinch the nostrils closed, ensure the mouth is closed, and reattempt to v
entilate.
D: hyperextend the patients head, insert an oral airway, and reattempt to ven
tilate.
Reason:
The patient has likely had a partial laryngectomy and is a partial neck breather.
Because these patients are able to inhale and exhale some air via the nose and m
outh, you should pinch the nostrils closed, ensure that the mouth is closed, and
reattempt to ventilate. If you are still unable to achieve visible chest rise,
suctioning of the stoma may be required. When ventilating a stoma patient, the h
ead should be kept in a neutral position with the shoulders slightly elevated. T
his position allows for more effective ventilation.
Answer is C
==== 66/115 Airway
A firefighter was exposed to smoke during a structure fire. He is conscious, ale
rt, and oriented, but is experiencing respiratory distress. His oxygen saturatio
n is 91% on room air and his heart rate is rapid and strong. Which of the follow
ing is the MOST appropriate initial means of oxygenating this patient?
A: Nasal cannula set at 3 L/min.
B: Simple face mask set at 10 L/min.
C: Bag-mask device with high-flow oxygen.
D: Nonrebreathing mask set at 12 to 15 L/min.
Reason:
A room air oxygen saturation reading of 91% indicates mild hypoxemia. Because th
e patient is conscious, alert, and oriented and is not showing outward signs of
inadequate breathing, the most appropriate initial means of providing oxygenatio
n involves applying a nonrebreathing mask with the flow rate set at 12 to 15 L/m
in. You must closely monitor the patient's breathing adequacy and be prepared to
assist his ventilations with a bag-mask device.
Answer is D
==== 67/115 Airway
Which of the following MOST accurately describes the process of gas exchange in
the lungs?
A: The gases exchanged in the lungs move from an area of greater concentrati
on to an area of lesser concentration.
B: Blood that returns to the lungs from the right side of the heart has a sl
ightly lower level of carbon dioxide than oxygen.
C: The partial pressure of oxygen in the alveoli is typically between 40 and
50 torr at the end of a maximal inhalation.
D: The transfer of carbon dioxide from the alveoli into the bloodstream is f
acilitated by a process called diffusion.
Reason:
Pulmonary (external) respiration is defined as the exchange of gases in the lung
s. Gases exchanged in the lungs (O2 and CO2) move from an area of greater concen
tration to an area of lesser concentration by a process called diffusion. Blood
that enters the lungs from the right side of the heart has a PO2 of approximatel
y 40 mm Hg and a PCO2 of approximately 46 mm Hg. Within the lungs, carbon dioxid
e diffuses from the bloodstream into the alveoli while oxygen diffuses from the
alveoli into the bloodstream. The partial pressure of oxygen within the alveoli
is near 100 mm Hg, while the partial pressure of carbon dioxide is near 0 mm Hg.
Answer is A
==== 68/115 Airway
A patient presents with an acute onset of dyspnea. Which of the following condit
ions would be the LEAST likely underlying cause?
A: Pulmonary embolism
B: Hyperventilation
C: Pneumothorax
D: Pneumonia
Reason:
Pneumonia, an intrapulmonary infection, typically presents with a productive cou
gh, fever and chills, and respiratory distress that gradually worsens. Hypervent
ilation, pulmonary embolism, and pneumothorax are all acute events, and thus pre
sent with acute dyspnea.
Answer is D
==== 69/115 Airway
Which of the following signs is unique to a severe foreign body upper airway obs
truction?
A: Tachycardia
B: Difficulty speaking
C: Anxiety
D: Perioral cyanosis
Reason:
Signs of a severe (complete) upper airway obstruction include a weak, ineffectiv
e cough; an inability to speak; and cyanosis, which indicates decreased blood ox
ygen levels. Tachycardia and anxiety can occur with both mild (partial) and seve
re airway obstructions. Signs of a mild foreign body airway obstruction include
a strong cough, difficulty speaking, normal skin color, and a normal level of co
nsciousness.
Answer is D
==== 70/115 Airway
Snoring respirations are MOST rapidly corrected by:
A: suctioning of the oropharynx.
B: placing the patient on his or her side.
C: manually maneuvering the head.
D: inserting a simple airway adjunct.
Reason:
Snoring (sonorous) respirations indicate partial airway obstruction by the tongu
e. The quickest way to correct this is to perform a head tilt-chin lift maneuver
, or jaw-thrust maneuver if trauma is suspected. If secretions are in the airway
, suction the oropharynx after manually opening the airway. A simple airway adju
nct (oral or nasal airway) should be inserted to assist in maintaining airway pa
tency after manually opening the airway and suctioning the oropharynx if needed.
Placing a patient in a recumbent (on their side) position is appropriate if he
or she has a decreased level of consciousness, is uninjured, and is breathing ad
equately.
Answer is C
==== 71/115 Airway
Which of the following is the LEAST reliable when assessing the oxygenation and/
or perfusion status of an adult patient?
A: Pulse oximetry
B: Skin condition
C: Mental status
D: Capillary refill
Reason:
Because peripheral perfusion decreases with age, capillary refill time is a less
reliable indicator of oxygenation and perfusion in adults; it is more reliable
in children younger than 6 years of age. It is important to remember that factor
s such as cold temperatures can affect capillary refill time. Skin condition and
mental status are excellent indicators of a person's oxygenation and perfusion
status, regardless of age. Although not without limitations (ie, abnormal hemogl
obin binding [CO toxicity]), pulse oximetry is a more reliable indicator of oxyg
enation than capillary refill time.
Answer is D
==== 72/115 Airway
Which of the following patients would benefit the MOST from continuous positive
airway pressure (CPAP)?
A: 61-year-old male with severe respiratory distress and diffuse crackles
B: 39-year-old female with a cough and scattered wheezing
C: 45-year-old male with anaphylaxis and severe upper airway swelling
D: 72-year-old unresponsive male with slow, irregular breathing
Reason:
Continuous positive airway pressure (CPAP) is a noninvasive form of positive-pre
ssure ventilation. It is used in the treatment of CHF with pulmonary edema, as e
vidence by respiratory distress and abnormal breath sounds (ie, rhonchi, crackle
s), and in patients with severe acute bronchospasm (ie, asthma). CPAP transmits
positive-pressure to the lower airways, where it forces fluid from the alveoli,
reexpands atelectatic alveoli, and opens the bronchioles. For acute bronchospasm
, CPAP is commonly used in conjunction with an in-line nebulizer, which simultan
eously delivers beta-2 agonist medications. Patients receive the maximum benefit
from CPAP during exhalation because they are breathing against a positive-press
ure gradient, which can be adjusted accordingly. CPAP has been shown to reduce t
he need for intubation and often causes rapid improvement in the patient's clini
cal status. CPAP is contraindicated in patients who are unable to follow verbal
commands and in those who are hypoventilating (eg, slow rate, reduced tidal volu
me).
Answer is A
==== 73/115 Airway
Which of the following assessment findings indicates a patent airway?
A: Loud inspiratory stridor and severe pallor
B: Diaphoresis and a forceful cough
C: Semiconscious with snoring respirations
D: Gurgling respirations and cyanosis
Reason:
A patent airway is one that is free of obstructions and secretions. Gurgling res
pirations indicate fluid in the airway. Snoring respirations indicate partial ob
struction by the tongue, the most common cause of airway obstruction in unrespon
sive patients. Inspiratory stridor indicates upper airway swelling. Cyanosis, al
though a clear sign of hypoxia, is a relatively later finding. A patient who is
coughing forcefully has a patent airway.
Answer is B
==== 74/115 Airway
You have inserted an oral airway in an apneic patient and are ventilating him wi
th a bag-mask device when he suddenly vomits. After removing the oral airway, yo
u should:
A: apply cricoid pressure.
B: insert a Combitube.
C: suction his oropharynx.
D: turn him onto his side.
Reason:
Keeping in mind that mortality increases significantly if aspiration occurs, you
must immediately remove the oral airway and turn the patient onto his sidewhich
will facilitate drainage of vomitus from his mouthand then suction his oropharynx
. Do not suction the unprotected airway of a patient while he or she is supine;
this only increases the likelihood of aspiration. Posterior cricoid pressure (Se
llick maneuver) is no longer a recommended technique as it has been shown to imp
air ventilation and does not prevent gastric distention and regurgitation as was
once thought. A Combitube or other airway device designed to enter the esophagu
s should not be inserted if the patient is actively vomiting.
Answer is D
==== 75/115 Airway
Which of the following is the MOST appropriate and effective method of oxygenati
ng a semiconscious patient with slow, shallow breathing?
A: Nonrebreathing mask at 15 L/min
B: Bag-mask device and high-flow oxygen
C: Oxygen-powered transport ventilator
D: Nasal cannula at 1 to 6 L/min
Reason:
Patients with slow, shallow (reduced tidal volume) breathing are not taking in s
ufficient amounts of air to maintain adequate minute volume and require some for
m of positive-pressure ventilation. This is especially true for patients who are
semiconscious or unconscious. Assist the patient's ventilations with a bag-mask
device attached to high-flow oxygen. Neither the nasal cannula nor the nonrebre
athing mask will improve the patient's tidal volume, and would therefore be of l
ittle benefit to the patient with inadequate ventilation. Transport ventilators
are used for unresponsive apneic patients, generally after they have been intuba
ted.
Answer is B
==== 76/115 Airway
A 60-year-old man was injured when his tractor rolled over on him. The tractor h
as been stabilized by rescue personnel. When you assess the man, you note that h
e is responsive to pain only. You should:
A: open his airway with the jaw-thrust maneuver.
B: assess his breathing for 5 to 10 seconds.
C: apply oxygen with a nonrebreathing mask.
D: begin some form of positive-pressure ventilation.
Reason:
This patient's level of consciousness is markedly diminished; you must ensure th
at his airway is open. The trauma patient's airway is opened by manually stabili
zing his or her head and using the jaw-thrust maneuver. However, if the jaw-thru
st maneuver does not adequately open the patient's airway, you should carefully
perform the head tilt-chin lift manuever. After his airway is open, ensure that
it is clear of secretion or foreign bodies; use suction as needed. After establi
shing a patent airway, assess the adequacy of his breathing and treat accordingl
y.
Answer is A
==== 77/115 Airway
Immediately after placing an endotracheal tube in the trachea of an adult, the p
aramedic should:
A: secure the tube with an appropriate device.
B: inflate the distal cuff with 5 to 10 mL of air.
C: attach the bag-mask device and ventilate.
D: attach an end-tidal carbon dioxide detector.
Reason:
Immediately following placement of an endotracheal tube, the paramedic should in
flate the distal cuff with 5 to 10 mL of air and detach the syringe. This is a c
ritical step in the intubation procedure because the distal cuff protects the lu
ngs from aspiration. After the cuff is inflated and the stylet has been removed,
attach an end-tidal CO2 detector (quantitative waveform capnography should be u
sed) in between the bag-mask device and ET tube, begin to ventilate the patient,
and auscultate over the epigastrium and the apices and bases of both lungs. Aft
er confirming proper ET tube placement, secure the tube with an appropriate devi
ce; a commercially-manufactured tube-securing device is recommended.
Answer is B
==== 78/115 Airway
When assessing a responsive patient with a suspected pulmonary embolism (PE), it
is important to remember that:
A: a PE is caused by a problem in breathing mechanics.
B: ventilation continues, but oxygenation is inadequate.
C: most conscious patients with a PE have cape cyanosis.
D: hypocarbia quickly develops due to hypoventilation.
Reason:
A pulmonary embolism (PE) occurs when a thrombus breaks free from another part o
f the body (often a deep vein thrombosis [DVT]) and lodges in a pulmonary artery
. As a result, pulmonary gas exchange, tissue oxygenation, and perfusion are imp
aired. However, the patient continues to ventilatethat is, he or she continues to
move air into and out of the lungs. This lack of oxygenation and perfusion, des
pite ongoing ventilation, creates a ventilation-perfusion mismatch. Conditions t
hat affect the mechanics of breathing include flail chest, rib fractures, and te
nsion pneumothorax, among others. Hypoventilation leads to hypercarbia (CO2 rete
ntion), not hypocarbia. Cape cyanosisprofound cyanosis to the upper torso, should
ers, and faceis often observed in patients with a massive PE, specifically a sadd
le embolus, which lodges where the main pulmonary artery branches into the left
and right pulmonary arteries. Most patients with a saddle embolus are in cardiac
arrest. Cape cyanosis is not commonly observed in responsive patients with a sm
aller PE.
Answer is B
==== 79/115 Airway
An elderly man is unresponsive and apneic. What should you do?
A: Assess for a carotid pulse.
B: Perform immediate intubation.
C: Assess his cardiac rhythm.
D: Provide two rescue breaths.
Reason:
After determining that an adult patient is unresponsive and apneic, you should a
ssess for a carotid pulse for at least 5 seconds but no more than 10 seconds. If
a pulse is present, provide rescue breathing (10 to 12 breaths/min in the adult
). If a pulse is absent, begin CPR (starting with chest compressions), then open
the airway and give two rescue breaths. Continue CPR and assess his cardiac rhy
thm as soon as possible. If you are able to effectively ventilate the patient wi
th an oral airway and bag-mask device, intubation should not be an immediate pri
ority. Furthermore, you should not attempt intubation until the patient has rece
ived at least 2 to 3 minutes of preoxygenation. Rare is the need to immediately
intubate anyone.
Answer is A
==== 80/115 Airway
When ventilating an intubated patient in cardiac arrest, which of the following
end-tidal CO2 (PETCO2) findings indicates return of spontaneous circulation?
A: Abrupt and sustained decrease in PETCO2
B: Abrupt and sustained increase in PETCO2
C: Progressive decrease in PETCO2
D: Complete absence of a PETCO2 reading
Reason:
During cardiac arrest and other low perfusion states, decreased amounts of carbo
n dioxide are returned to the lungs due to anaerobic metabolism and lactic acido
sis. This explains why you typically see progressively decreasing end-tidal CO2
(PETCO2) readings in cardiac arrest patients. However, if return of spontaneous
circulation (ROSC) occurs, increased amounts of carbon dioxide are returned to t
he lungs, resulting in an abrupt and sustained increased in PETCO2 that is typic
ally equal to or greater than 40 mm Hg. A complete absence of a PETCO2 reading a
nd capnographic waveform indicates that the endotracheal tube is not in the trac
hea.
Answer is B
==== 81/115 Airway
When assessing an unresponsive patient, you note that he is not breathing. Which
of the following airway devices or interventions is contraindicated?
A: Laryngeal mask airway
B: Nasotracheal intubation
C: Esophageal Combitube
D: Orotracheal intubation
Reason:
Because the endotracheal tube is advanced when the patient inhales (the vocal co
rds are open at their widest during inhalation), blind nasotracheal intubation i
s contraindicated in apneic patients. Orotracheal intubation, as well as alterna
tive airway devices (ie, Combitube, laryngeal mask airway [LMA], King LT airway)
, would be appropriate devices used to secure the patient's airway, provided tha
t he does not have an intact gag reflex.
Answer is B
==== 82/115 Airway
Which of the following would MOST likely cause laryngeal spasm?
A: Insertion of a laryngeal mask airway
B: Forceful positive-pressure ventilation
C: Inhalation of cool, humidified mist
D: Extubation of a semiconscious patient
Reason:
Laryngeal spasmspasmodic closure of the vocal cordsis often caused by an overly ag
gressive intubation technique. It may also occur during extubation, especially i
f the patient is semiconscious (one of the many reasons to avoid field extubatio
n!). Cool, humidified mist is often used to reduce upper airway edema, such as t
hat caused by croup. Insertion of the laryngeal mask airway (LMA) is not associa
ted with a high incidence of laryngeal spasm because the device is not placed in
between the vocal cords. The most significant complications of forceful positiv
e-pressure ventilation include barotrauma, which may cause a pneumothorax; gastr
ic distention, which may cause regurgitation and aspiration in the nonintubated
patient; and reduced venous return to the heart (preload), which may cause hypot
ension.
Answer is D
==== 83/115 Airway
You are ventilating an apneic, intubated patient and note that his ETCO2 reading
, per waveform capnography, is 56 mm Hg. You should:
A: decrease the rate of ventilations.
B: attach a colorimetric ETCO2 detector.
C: extubate the patient immediately.
D: increase the rate of ventilations.
Reason:
When ventilating an apneic patient with spontaneous circulation (eg, a pulse), y
our goal is to maintain an end-tidal CO2 (ETCO2) reading of 35 to 45 mm Hg. An i
ncreasing ETCO2 indicates excess CO2 in the patients exhaled air, and should be t
reated by increasing the rate of your ventilations. A low ETCO2 readingagain, in
an apneic patient with spontaneous circulationindicates a low amount of CO2 in ex
haled air; therefore, you should slow your rate of ventilations accordingly. The
sudden loss of a capnographic waveform and LED reading indicates that the ET tu
be has become dislodged and is no longer in the trachea. If inadvertent extubati
on occurs, remove the ET tube immediately and resume bag-mask ventilations. The
colorimetric ETCO2 detector is a qualitative device that simply indicates the pr
esence of carbon dioxide during exhalation; it is less reliable than quantitativ
e waveform capnography.
Answer is D
==== 84/115 Airway
Which of the following occurs when a patient is hypoventilating?
A: Decreased PaCO2, increased PaO2, increased pH
B: Increased PaO2, increased PaCO2, decreased pH
C: Decreased PaO2, decreased PaCO2, increased pH
D: Increased PaCO2, decreased PaO2, decreased pH
Reason:
Hypoventilation, by definition, is a condition in which the body does not elimin
ate adequate amounts of carbon dioxide. This is typically the result of inadequa
te ventilation (eg, reduced tidal volume, bradypnea, etc.). Because the hypovent
ilating patient is retaining carbon dioxide, the PaCO2 would increase and the pH
of the blood and CSF would decrease (acidosis). Insufficient carbon dioxide eli
mination is accompanied by insufficient oxygen intake; therefore, the PaO2 would
decrease.
Answer is D
==== 85/115 Airway
You are transporting a woman with a history of COPD, who called EMS when her dif
ficulty breathing suddenly worsened. She is receiving oxygen at 4 L/min via nasa
l cannula, is on a cardiac monitor, and has an IV line in place. During your rea
ssessment, you note that she is responsive to pain only, is increasingly tachyca
rdic, and is developing cyanosis around her mouth. You should:
A: place her on her side in case she vomits.
B: insert a Combitube to protect her airway.
C: apply a nonrebreathing mask at 15 L/min.
D: assist her breathing with a bag-mask device.
Reason:
A decreased level of consciousness, tachycardia, and cyanosis indicate that your
patient is no longer breathing adequately and has worsened hypoxemia. You shoul
d begin assisting her breathing with a bag-mask device and 100% oxygen. Patients
with inadequate breathing (ie, shallow breathing [reduced tidal volume], respir
ations that are too fast or too slow) need some form of positive-pressure ventil
ation to maintain adequate minute volume; this cannot be accomplished with a non
rebreathing mask. If the patients level of consciousness deteriorates to a point
of unresponsiveness, advanced airway management (ie, endotracheal intubation, Co
mbitube, CobraPLA) should be considered. The recovery position is appropriate fo
r unresponsive patients with adequate breathing; this patient is not breathing a
dequately.
Answer is D
==== 86/115 Airway
When administering oxygen via nasal cannula during a long-range transport, you s
hould:
A: set the flow rate to at least 4 L/min.
B: ensure that the patient is supine.
C: attach an oxygen humidifier.
D: vary the liter flow from 1 to 6 L/min.
Reason:
Oxygen that is delivered nasally, especially over a prolonged period of time, ca
n cause drying and irritation of the nasal mucosa; therefore, an oxygen humidifi
er should be attached. The appropriate oxygen flow rate for the nasal cannula is
1 to 6 L/min. A semi-sitting (semi-Fowler) position is preferable for most pati
ents, especially those who are experiencing breathing difficulty.
Answer C
==== 87/115 Airway
A 40-year-old male presents with acute respiratory distress while eating a meal.
He is conscious and alert; is able to speak, but with difficulty; and has pink,
moist skin. Which of the following statements BEST describes this patients condi
tion?
A: Mild airway obstruction with adequate air exchange
B: Diffuse bronchospasm with good air exchange
C: Progressive swelling of the upper airway structures
D: Severe airway obstruction with poor air exchange
Reason:
The patient in this scenario has a mild (partial) foreign body airway obstructio
n and is exchanging air adequately. He is conscious and alert; able to talk, alt
hough with difficulty; and is not cyanotic. Patients with a severe (complete) ai
rway obstruction are not able to cough, talk, or breathe. They quickly become hy
poxic, which manifests with a decreased level of consciousness and cyanosis. Giv
en the circumstances in which the patients acute difficulty breathing beganand the
fact that he is not wheezingbronchospasm is unlikely. Progressive upper airway s
welling would be just thatprogressive; this patients difficulty breathing began su
ddenly. Furthermore, upper airway swelling would produce inspiratory stridor, wh
ich he does not have.
Answer is A
==== 88/115 Airway
A patient with a reactive lower airway disease would be expected to present with
:
A: severe hypocarbia.
B: pleuritic chest pain.
C: inspiratory stridor.
D: expiratory wheezing.
Reason:
Reactive lower airway disease includes any condition associated with bronchospas
m (ie, asthma, bronchiolitis). A hallmark sign of bronchospasm is wheezinga high-
pitched whistling sound that indicates air movement through narrowed bronchioles
. Stridor is a high-pitched sound heard during inhalation and indicates obstruct
ion of the upper airway due to swelling or a foreign body. Because bronchospasm
impairs pulmonary respiration, the patient would become hypercarbic, not hypocar
bic. Pleuritic (sharp) chest pain is common in patients with a spontaneous pneum
othorax or pulmonary embolism; it is not a common finding in patients with react
ive airway disease.
Answer is D
==== 89/115 Airway
What is the approximate minute alveolar volume of a patient who is breathing at
a rate of 26 breaths/min with an estimated tidal volume of 450 mL?
A: 10,100 mL
B: 11,700 mL
C: 8,200 mL
D: 9,400 mL
Reason:
Minute alveolar volume, also called minute alveolar ventilation, is the volume o
f air that reaches the alveoli and participates in pulmonary gas exchange each m
inute. It is computed by multiplying the patient's tidal volumeless the dead spac
e volumeand the patient's respiratory rate. Dead space volumethe volume of air tha
t lingers in the upper airway and does not reach the alveoliis approximately 30%
(or 1 mL per pound of body weight) of the patient's tidal volume. Thus, if the p
atient's tidal volume is 450 mL, the actual volume of air that enters the lungs
per breath is approximately 315 mL (450 mL - 135 mL [30% of 450 mL] = 315 mL). T
herefore, the patient's minute alveolar volume is approximately 8,200 mL (315 mL
x 26 breaths/min = 8,190 mL).
Answer is C
==== 90/115 Airway
Prior to applying a nonrebreathing mask on a responsive patient with respiratory
distress, you should:
A: set the oxygen flowmeter to no more than 10 L/min.
B: ask the patient to exhale fully and then hold his breath.
C: ensure that the reservoir bag is fully inflated.
D: place the patient in a lateral recumbent position.
Reason:
Prior to placing a nonrebreathing mask on a patient, you must ensure that the re
servoir bag is completely filled. If it is not prefilled, the nonrebreathing mas
k will not deliver high concentrations of oxygen. The appropriate oxygen flow ra
te for a nonrebreathing mask is 12 to 15 L/min. A responsive patient with adequa
te breathing and no trauma should be placed in a position of comfort. A recumben
t position is necessary if the patient is unresponsive, is not injured, and is b
reathing adequately.
Answer is C
==== 91/115 Airway
Which of the following is associated with insufficient carbon dioxide eliminatio
n?
A: A venous PCO2 of 46 torr.
B: Acute hyperventilation
C: Bradypnea and hypopnea
D: Tachypnea and hyperpnea
Reason:
Hypopnea (shallow breathing [reduced tidal volume]) and bradypnea (slow respirat
ions) result in both insufficient oxygen intake and insufficient carbon dioxide
(CO2) elimination and should be treated with positive-presure ventilation to inc
rease oxygen intake and facilitate CO2 elimination. A patient with tachypnea (ra
pid breathing) and hyperpnea (deep breathing) eliminates adequateoften increasedam
ounts of CO2 from the body. Hyperventilation occurs when CO2 elimination exceeds
CO2 production. A venous PCO2 of 46 torr represents a normal level, and thus, a
dequate CO2 elimination.
Answer is C
==== 92/115 Airway
You are ventilating an unresponsive, apneic 50-year-old man. He has a pulse, but
it is rapid and weak. When ventilating this patient, it is MOST important to:
A: deliver one breath every 3 seconds.
B: provide 8 to 10 breaths per minute.
C: ventilate until the chest visibly rises.
D: promptly insert an endotracheal tube.
Reason:
When ventilating any apneic patient, you should deliver each breath over a perio
d of 1 secondjust enough to produce visible chest rise. You should avoid hyperven
tilating the patient as this increases the incidence of gastric distention and t
he associated risks of regurgitation and aspiration; it may also impede venous r
eturn to the heart secondary to increased intrathoracic pressure. In the apneic
adult with a pulse, deliver one breath every 5 to 6 seconds (10 to 12 breaths/mi
n). If the patient (adult, child, or infant) is apneic and pulseless, ventilatio
ns should be provided at a rate of 8 to 10 breaths/min (one breath every 6 to 8
seconds) after an advanced airway device has been inserted. Endotracheal intubat
ion is not always an immediate priority, especially if you are able to maintain
a patient's airway with manual positioning and a simple adjunct and are able to
adequately ventilate him or her with a bag-mask device.
Answer is C
==== 93/115 Airway
In which of the following situations would it be MOST appropriate to insert a Ki
ng LT supraglottic airway?
A: Unresponsiveness following oven cleaner ingestion
B: Comatose patient who ingested aspirin
C: Unresponsive patient with a minimal gag reflex
D: Apneic patient with cancer of the esophagus
Reason:
The King LT is a supraglottic airway device that can be used as an alternative t
o endotracheal intubation or if endotracheal intubation has been attempted witho
ut success. It has been shown to provide better ventilation than the bag-mask de
vice. The King LT is advantageous in that laryngoscopy and visualization of the
vocal cords are not required, and the insertion technique requires less training
than endotracheal intubation. The King LT is only used in unresponsive patients
; it is contraindicated in those with an intact gag reflex (even if the gag refl
ex is minimal), in patients who ingested a caustic substance (eg, oven cleaner [
strong acid]), and in patients with known esophageal disease (eg, cancer, varice
s).
Answer is B
==== 94/115
In an otherwise healthy individual, breathing is primarily stimulated by:
A: an increase in arterial CO2.
B: an increase in arterial O2.
C: a decrease in arterial CO2.
D: a decrease in arterial O2.
Reason:
The primary stimulus to breathe in an otherwise healthy individual is an increas
e in the level of arterial carbon dioxide and a decrease in the pH of the cerebr
ospinal fluid (CSF). A decreased level of arterial oxygen is also a powerful sti
mulus to breathe (hypoxic drive), but is not the primary stimulus in otherwise h
ealthy individuals.
Answer is A
==== 95/115 Airway
You are assisting the ventilations of an adult with a bag-mask device. The patie
nt is responsive to pain only and has a heart rate of 140 beats/min. Which of th
e following signs would indicate that your assisted ventilations are inadequate?
A: Equal chest wall excursion
B: Increased ventilation compliance
C: A marked increase in heart rate
D: Minimal abdominal movement
Reason:
Signs of inadequate artificial ventilation include minimal or no rise of the che
st, significant air leakage from around the mask, an increase in the patient's h
eart rate, cyanosis that is not resolving, a falling oxygen saturation, and decr
eased ventilation compliance (increased resistance when ventilating). Tachycardi
a is a sign of hypoxia and should begin to resolve if your assisted ventilations
with high-flow oxygen are adequate.
Answer is C
==== 96/115 Airway
A COPD patient presents with severe respiratory distress; cyanosis to the face,
neck, and chest; a decreased level of consciousness; and a heart rate of 150 bea
ts/min. The MOST appropriate treatment for this patient includes:
A: oxygen via nasal cannula and 6 mg of adenosine IV.
B: hyperventilation followed by orotracheal intubation.
C: high-flow oxygen via nonrebreathing mask.
D: assisting ventilations with a bag-mask device.
Reason:
The patient in this scenario is experiencing respiratory failure and severe hypo
xia and requires immediate positive-pressure ventilatory assistance. Initially,
this should be provided with a bag-mask device attached to 100% oxygen. Devices
such as a nonrebreathing mask and nasal cannula deliver oxygen passively, not vi
a positive-pressure. Patients with inadequate ventilation benefit very little fr
om passive oxygenation. The patient may require intubation, but not before adequ
ate preoxygenation with basic means. Do NOT hyperventilate the patient as this c
auses an increase in intrathoracic pressure. Because of chronic air trapping in
the lungs, COPD patients are especially susceptible to pneumothoraces caused by
barotrauma. Furthermore, hyperventilation impairs venous return, which can compr
omise cardiac output. Adenosine is not indicated; the patients tachycardia is lik
ely a compensatory response to severe hypoxia, not a primary cardiac dysrhythmia
.
Answer is D
==== 97/115 Airway ***REPEAT***
Immediate treatment for an unresponsive patient with sonorous breathing involves
:
A: orotracheal intubation.
B: manual head positioning.
C: oropharyngeal suctioning.
D: inserting an oral airway.
Reason:
Sonorous (snoring) breathing indicates partial blockage of the airway by the ton
gue. This is the most common cause of anatomic upper airway obstruction in patie
nts with a decreased level of consciousness. The quickest way to correct this is
to manually position the head (ie, head tilt-chin lift, jaw-thrust). Then, an o
ral or nasal airway should be inserted, which, in conjunction with manual head p
ositioning, helps maintain patency of the airway. Intubation is indicated for pa
tients who are unresponsive and unable to maintain their own airway; however, it
should not precede basic airway maneuvers (eg, manual head positioning, basic a
irway adjunct) and preoxygenation with a bag-mask device. If secretions are jeop
ardizing a patients airway, as evidenced by gurgling respirations, the oropharynx
must be suctioned.
Answer is B
==== 98/115 Airway
You are ventilating an intubated patient who has been in cardiac arrest for appr
oximately 15 minutes. Despite the presence of bilaterally equal breath sounds, q
uantitative capnography persistently reads less than 10 mm Hg. This MOST likely
indicates that:
A: you are ventilating the patient too fast.
B: you need to increase the ventilatory rate.
C: the patient is not producing carbon dioxide.
D: the bodys cells are rapidly metabolizing oxygen.
Reason:
Before carbon dioxide can be eliminated from the body and detected with capnogra
phy, the body must be able to metabolize oxygen in order to produce carbon dioxi
de (aerobic metabolism). Patients in cardiac arrestespecially prolonged cardiac a
rrestare severely acidotic; the cells are metabolizing carbon dioxide and produci
ng lactic acid (anaerobic metabolism). As a result, minimal (or no) carbon dioxi
de is being produced and returned to the lungs. This would result in a persisten
tly low (< 10 mm Hg) end-tidal CO2 (ETCO2) reading. However, when ventilating an
intubated patient who has a pulsethat is, he or she is perfusing and is producin
g carbon dioxidea low (< 35 mm Hg) ETCO2 reading indicates that you are ventilati
ng too fast and are eliminating too much CO2 from the body. Therefore, you shoul
d decrease the rate of ventilation. By contrast, if the intubated patient with a
pulse displays an ETCO2 reading that is too high (> 45 mm Hg), he or she has ex
cess carbon dioxide that needs to be eliminated; therefore, you should increase
the rate of ventilation.
Answer is C
==== 99/115 airway
The exchange of oxygen and carbon dioxide between inspired air and the pulmonary
capillaries is called:
A: internal respiration.
B: intrapulmonary shunting.
C: pulmonary ventilation.
D: external respiration.
Reason:
Respiration is defined as the exchange of oxygen and carbon dioxide between the
body and its environment. External (pulmonary) respiration is the exchange of ox
ygen and carbon dioxide between inspired air and the pulmonary capillaries. Inte
rnal (cellular) respiration is the transfer of oxygen and carbon dioxide between
the capillaries and tissue cells. Pulmonary ventilation is defined as the movem
ent of air into and out of the lungs. Intrapulmonary shunting is an abnormal pro
cess in which external respiration is impaired due to alveolar dysfunction or da
mage (eg, atelectasis, pulmonary edema). Deoxygenated blood from the right side
of the heart bypasses the nonfunctional alveoli and returns to the left side of
the heart, still deoxygenated. The more diffuse the alveolar dysfunction, the gr
eater the degree of intrapulmonary shunting.
Answer is D
==== 100/115 Airway
You have been ventilating an unresponsive apneic 42-year-old male for approximat
ely 12 minutes. After securing his airway with an endotracheal tube and confirmi
ng proper ET tube placement, you should:
A: deliver one breath every 6 to 8 seconds.
B: insert an orogastric or nasogastric tube.
C: maintain an end-tidal CO2 of 45 to 50 mm Hg.
D: provide a tidal volume of about 10 mL/kg.
Reason:
Prolonged bag-mask ventilations often result in gastric distention. After the pa
tient has been intubated, excessive air in the stomach may impede your ability t
o deliver adequate tidal volume. Therefore, you should consider inserting an oro
- or nasogastric tube after the ET tube has been placed and proper positioning o
f the tube has been confirmed. When ventilating an apneic adult who has a pulse,
deliver one breath every 5 to 6 seconds (10 to 12 breaths/min). A ventilation r
ate of 8 to 10 breaths/min (one breath every 6 to 8 seconds) is appropriate for
patients in cardiac arrest, after an advanced airway device has been placed. Nor
mal tidal volume for the average adult male is 5 to 7 mL/kg (about 500 mL); a ti
dal volume of 10 mL/kg is clearly too much volume and may result in barotrauma.
Remember to ventilate the patient over 1 secondjust enough to produce visible che
st rise. Capnography should be used in conjunction with intubation to quantify a
dequate CO2 elimination. In the apneic patient with a pulse, you should maintain
an end-tidal CO2 (ETCO2) between 35 and 45 mm Hg. An ETCO2 reading greater than
45 mm Hg in an intubated apneic patient with a pulse usually indicates that you
are ventilating too slowly.
Answer is B
==== 101/115 Airway
Which of the following clinical presentations is MOST indicative of a patent air
way?
A: Perioral cyanosis; weak stridor
B: Pallor; confusion; audible wheezing
C: Diaphoresis; tachycardia; gagging
D: Flushed, moist skin; weak cough
Reason:
A patent (open) airway is one that does not impede airflow into and out of the l
ungs. Gagging is a forceful muscular contraction of the pharyngeal muscles and t
he glottis. This reaction is automatic when something touches an area deep in th
e oral cavitythat is, when the gag reflex is stimulated. The presence of gagging
indicates an intact gag reflex. Coughing is a forceful exhalation produced with
a greater than normal volume of air. A person with a weak cough, which could ind
icate an airway obstruction or suppression of the cough reflex by drugs or traum
a, is at serious risk of aspiration. Weak stridor is a sign of a marked reductio
n in airflow due to severe swelling of the upper airway. Cyanosis and weak strid
or clearly indicate a jeopardized airway. Audible wheezingthat is, wheezing that
can be heard without a stethoscopeindicates airflow obstruction in the lower airw
ay. In the presence of an altered mental status, audible wheezing indicates sign
ificant hypoxemia.
Answer is C
==== 102/115 Airway
Ventilation of an adult patient with a stoma and no tracheostomy tube is MOST ef
fectively achieved by:
A: hyperextending the patients head and ventilating with a pocket face mask.
B: using an infant- or child-size mask attached to an adult-size bag-mask de
vice.
C: ventilating with a manually triggered ventilator attached to an adult-siz
e mask.
D: using a child-size bag-mask device and sealing the patients mouth and nose
.
Reason:
Ventilation of the stoma patient does not require manual head positioning (eg, h
ead tilt-chin lift, jaw thrust); it can be performed with the patients head in a
neutral position. If the patient has a stoma and no tracheostomy tube, ventilati
ons can be performed using the mouth-to-stoma (with a resuscitation mask) techni
que or with a bag-mask device. Regardless of the technique used, you should use
an infant- or child-size mask to facilitate an adequate seal over the stoma. Usi
ng an infant- or child-size bag-mask device to ventilate an adult patient with a
stoma would likely not provide adequate tidal volume.
Answer is B
==== 103/115 Airway
Which of the following respiratory diseases is associated with decreased alveola
r elasticity, air trapping in the lungs, and an increase in residual lung volume
?
A: Asthma
B: Bronchiectasis
C: Chronic bronchitis
D: Emphysema
Reason:
Emphysema is characterized by destructive changes in the alveolar walls, includi
ng alveolar collapse and decreased alveolar elasticity. As a result, air becomes
trapped in the lungs and residual lung volume increases; this explains why pati
ents with emphysema have a barrel-shaped chest. Chronic bronchitis is caused by
an increase in the number and size of mucus-producing cells (Goblet cells), resu
lting in chronic inflammation and excessive mucus production in the bronchial tr
ee. Unlike emphysema, however, the alveoli generally are not seriously affected.
Emphysema and chronic bronchitis are both chronic obstructive pulmonary disease
s (COPD), and are most often the result of long-term cigarette smoking. Bronchie
ctasis is a disease that causes localized, irreversible dilation of part of the
bronchial tree; it is caused by a pus-producing bacterial infection of the bronc
hial wall. In bronchiectasis, affected bronchi are easily collapsible, which res
ults in impairment of airflow and clearance of pulmonary secretions. Because pat
ients with chronic bronchitis experience frequent respiratory infections, bronch
iectasis is especially common in this patient population. Asthmaa reactive airway
diseaseis a reversible condition characterized by contraction of the bronchiole
smooth muscle (bronchospasm), inflammation of the bronchial walls, and mucus plu
gging, which results in impaired airflow through the bronchioles. Common trigger
s to asthma include stress, temperature changes, and respiratory irritants (eg,
cigarette smoke). Asthma is also an obstructive lung disease; however, unlike em
physema and chronic bronchitis, it is episodic rather than chronic.
Answer is D
==== 104/115 Airway
You are en route to the hospital with a patient in respiratory extremis. You hav
e administered a sedative and a paralytic drug to the patient, but have been una
ble to successfully intubate him after two attempts. Your EMT partner tells you
that the patient's oxygen saturation is 98% and that his heart rate is 70 beats/
min. Your estimated time of arrival at the hospital is 7 minutes. Your MOST appr
opriate next action should be to:
A: resume bag-mask ventilations at 12 breaths/min and insert a multilumen or
a supraglottic airway device.
B: insert a Combitube to secure the patients airway and ventilate him at a ra
te of 20 to 24 breaths/min.
C: ask your partner to resume bag-mask ventilations as you prepare to perfor
m a surgical cricothyrotomy.
D: preoxygenate the patient for approximately 30 seconds and reattempt to su
ccessfully intubate the patient.
Reason:
Although your intubation attempts have been unsuccessful, the patients heart rate
and oxygen saturation are stable. If endotracheal intubation is unsuccessful, y
ou must always resume ventilations with a bag-mask device and 100% oxygen; inser
t an oral airway to help maintain patency of the airway. Because you are only 7
minutes away from the hospital, it would be quicker to insert a multilumen (eg,
Combitube) or supraglottic (eg, King LT, CobraPLA, LMA) airway device, after reo
xygenating him with a bag-mask device and 100% oxygen. Do NOT hyperventilate the
patient; ventilate him at a rate of 10 to 12 breaths/min. If you cannot adequat
ely ventilate the patient with a bag-mask device and are unable to insert a mult
ilumen or supraglottic airway device, you should proceed with a cricothyrotomy.
Answer is A
==== 105/115 Airway
During your assessment a patient with labored breathing, you note asymmetric che
st wall movement. This indicates that:
A: several ribs are fractured in several places.
B: one lung is inundated with fluid.
C: bilateral bronchospasm is present.
D: airflow into one lung is reduced.
Reason:
Asymmetric chest movementwhen one side of the chest (hemithorax) moves less than
the otherindicates that airflow into one lung is reduced. It could be the result
of conditions such as a tension pneumothorax or excess fluid in one lung (ie, un
ilateral pleural effusion). Bilateral bronchospasm results in decreased air move
ment in both lungs, and would cause both sides of the chest to move minimally. W
hen several ribs are fractured in more than one place, a free-floating segment o
f fractured ribs is created. This free-floating (flail) segmentnot necessarily th
e entire hemithoraxcollapses during inhalation and bulges during exhalation (para
doxical chest movement).
Answer is D
==== 106/115 Airway
With regard to endotracheal intubation, which of the following is the MOST harmf
ul to your patient if unrecognized?
A: Tracheal necrosis
B: Esophageal intubation
C: Barotrauma
D: Oral hemorrhage
Reason:
When inserting an endotracheal (ET) tube, nothing will kill your patient quicker
than inadvertently intubating the esophagus and not recognizing it! Not only mu
st you visualize the ET tube passing between the vocal cords, you must also use
additional methods of confirmation to ensure that the ET tube is in the trachea
(ie, auscultation of breath sounds, quantitative waveform capnography). Barotrau
ma, oral hemorrhage, and tracheal necrosis are potential complications associate
d with endotracheal intubation and can cause harm to your patient. However, fail
ure to ventilate the patient because the ET tube is in the esophagus is clearly
the most lethal.
Answer is B
==== 107/115 Airway
You are ventilating a patient with massive maxillofacial trauma after having suc
cessfully performed a needle cricothyrotomy. It is MOST important that you:
A: monitor the patients cardiac rhythm and oxygen saturation for signs of cat
heter dislodgement.
B: ventilate the patient for approximately 3 to 5 minutes before attempting
tracheal intubation.
C: open the release valve on the jet ventilator device just until the patien
ts chest visibly rises.
D: slightly extend the patients head to ensure consistent delivery of adequat
e ventilations.
Reason:
Cricothyrotomyneedle or surgicalis indicated when you are unable to ventilate a pa
tient by other, less invasive means (eg, bag-mask device, intubation). If you ar
e able to adequately ventilate the patient after performing a needle cricothyrot
omy, continue to do so and rapidly transport; you have achieved your objective.
As with any other technique of ventilation, you must ensure the chest rises adeq
uately with each ventilation. After performing a needle cricothyrotomy, this inv
olves opening the release valve on the jet ventilator just until the chest visib
ly rises. Extreme care must be exercised when providing positive-pressure ventil
ation to any patient. Overinflation of the lungs can cause barotrauma and a resu
ltant pneumothorax. An advantage of performing a cricothyrotomy is that it does
not require manipulation of the cervical spine. This is especially beneficial in
patients with massive maxillofacial trauma, in which case a cervical spine inju
ry should be assumed. Cardiac rhythm and oxygen saturation monitoring are import
ant when ventilating any patientregardless of the method of ventilation. However,
dislodgement of an advanced airway device would be detected quicker by monitori
ng for bilaterally equal breath sounds and using capnography. Do not rely solely
on cardiac monitoring and pulse oximetry.
Answer is A
==== 108/115 Airway
Which of the following patients is the BEST candidate for nasotracheal intubatio
n?
A: Unresponsive, mandibular fracture, apnea
B: Cerebrospinal rhinorrhea, semiconscious, bradypnea
C: Restless, audible wheezing, low oxygen saturation
D: Semiconscious, pulmonary edema, tachypnea
Reason:
Nasotracheal intubation is indicated for patients who are breathing spontaneousl
y, but require definitive airway management to prevent further deterioration of
their condition. Conscious patients or patients with an altered mental status an
d an intact gag reflex, who are in respiratory failure due to conditions such as
COPD, asthma, or pulmonary edema, are typical candidates for nasotracheal intub
ation. Because the tracheal tube is advanced during inhalationthe point at which
the glottis is open the widestnasotracheal intubation is contraindicated in apnei
c patients. It is also contraindicated in patients with nasal trauma or evidence
of a cribriform plate fracture (ie, cerebrospinal rhinorrhea). A restless patie
nt with audible wheezing and a low oxygen saturation should be treated initially
with supplemental oxygen and a beta-2 agonist drug (ie, albuterol) or a parasym
pathetic bronchodilator (ie, Atrovent). If the patients condition is refractory t
o supplemental oxygen and several bronchodilator treatments, and he or she is st
ill breathing spontaneously, nasotracheal intubation would be a consideration.
Answer is D
==== 109/115 Airway
A 66-year-old male presents with labored breathing. He is conscious and alert; h
owever, he is tachypneic, tachycardic, and is experiencing difficulty speaking.
He has a history of hypertension and congestive heart failure. Auscultation of h
is lungs reveals diffuse coarse crackles and his oxygen saturation is 86%. Which
of the following interventions will be of MOST benefit to this patient?
A: Ventilation with a bag-mask device
B: Sedation, chemical paralysis, and intubation
C: High-flow oxygen via nonrebreathing mask
D: Continuous positive airway pressure
Reason:
Continuous positive airway pressure (CPAP), a method of transmitting positive pr
essure into the lungs of a spontaneously breathing patient, is used in conjuncti
on with positive-end expiratory pressure (PEEP), which is set between 2.5 and 10
cm H2O. CPAP causes the patient to exhale against positive pressure; this reexp
ands collapsed alveoli and forces fluid from the alveoli. CPAP is useful in trea
ting patients with diffuse atelectasis and pulmonary edema; it reduces the work
of breathing and improves pulmonary respiration. Most patients who are placed on
CPAP are anxious initially; however, with an effective mask-to-face seal and re
assurance, clinical improvement is often noted rather quickly. Signs of improvem
ent include decreased work of breathing, increased ease in speaking, increasing
Sp02, and decreases in heart rate and respiratory rate. When used in conjunction
with medications to treat pulmonary edema (eg, nitroglycerin), CPAP has been sh
own to reduce the need for intubation. If CPAP fails, however, you must be able
to recognize patient deterioration and be prepared to assist ventilations with a
bag-mask device. CPAP is contraindicated in patients with slow, shallow (reduce
d tidal volume) breathing and in patients who are unable to follow simple comman
ds; these patients require assisted ventilation with a bag-mask device and possi
bly intubation. High-flow (12 to 15 L/min) oxygen via nonrebreathing mask increa
ses the fraction of inspired oxygen (FiO2); however, it is most effective if the
re are no barriers to diffusion in the lungs (ie, atelectasis, pulmonary edema).
Answer is D
==== 110/115 Airway
All of the following medications are used to chemically paralyze a patient to faci
litate endotracheal intubation, EXCEPT:
A: Pavulon.
B: Norcuron.
C: rocuronium.
D: etomidate.
Reason
Etomidate (Amidate) is a nonnarcotic, nonbarbiturate, sedative-hypnotic drug. It
is not a neuromuscular blocking agentthat is, it does not induce paralysis. Seda
tive-hypnotic drugs are used to sedate the patient before administering a neurom
uscular blocking agent (paralytic) and performing orotracheal intubation. Pancur
onium bromide (Pavulon), vecuronium bromide (Norcuron), and rocuronium bromide (
Zemuron) are all examples of neuromuscular blocking agents that induce chemical
paralysis; they all function at the neuromuscular junction and relax skeletal (s
triated) muscle by impeding the action of acetylcholine.
Answer is D
==== 111/115 Airway
A 60-year-old female presents with acute respiratory distress. The patient has a
tracheostomy tube in place, but is able to breathe spontaneously. She is consci
ous, but restless. Her heart rate is 120 beats/min and her oxygen saturation is
82%. You should:
A: attach a bag device to the tracheostomy tube and mildly hyperventilate he
r.
B: assess for secretions in the tracheostomy tube and suction the tube if ne
eded.
C: carefully remove the tracheostomy tube and replace it with a new one.
D: place an oxygen mask over the tracheostomy tube and reassess the patient.
Reason:
Acute respiratory distress in the patient with a tracheostomy tube is often the
result of thick secretions or a mucous plug in the tube. Therefore, you should i
mmediately assess the tracheostomy tube and determine if suctioning is required.
Never routinely remove a tracheostomy tube; there is no guarantee that you will
be able to replace it! If the patients clinical condition does not improve after
suctioning the tracheostomy tubeespecially if signs of inadequate ventilation ar
e noted (eg, reduced tidal volume, rapid or slow rate)you should assist the patie
nts ventilations; however, you should not hyperventilate. Hyperventilation impede
s venous return to the heart (preload) secondary to increased intrathoracic pres
sure. Placing an oxygen mask over the tracheostomy tube is appropriate only afte
r you have ensured that the tube is not obstructed and the patient is breathing
adequately.
Answer is B
==== 112/115 Airway
Which of the following is MOST consistent with a patient who is hypoventilating?
A: PO2 of 92 mm Hg
B: Rapid, thready pulse
C: pH of 7.49
D: PCO2 of 52 mm Hg
Reason:
Respiratory acidosis is always caused by hypoventilation. A patient who is hypov
entilating is retaining carbon dioxide and is not bringing in enough oxygen. The
refore, the PCO2 increases (> 45 mm Hg) and the PO2 decreases (< 80 mm Hg). If t
he body cannot intake adequate amounts of oxygen, tissue oxygenation will ultima
tely suffer. As a result, the cells convert to anaerobic metabolism, which produ
ces lactic acid and drives the pH down (< 7.35). A pH greater than 7.45 is consi
stent with metabolic alkalosis (ie, large quantities of ingested antacid) or res
piratory alkalosis (ie, hyperventilation). Tachycardia may be present with both
hypo- and hyperventilation.
Answer is D
==== 113/115 Airway
In contrast to the pneuomotaxic center of the medulla, the apneustic center:
A: decreases its influence in times of increased demand by increasing the re
spiratory rate.
B: is responsible for setting a persons resting respiratory rate.
C: influences the respiratory rate by increasing the number of inspirations
per minute.
D: is responsible for terminating inspiration to prevent overexpansion of th
e lungs.
Reason:
The respiratory center in the medulla is divided into three regions: the respira
tory rhythmicity center, the apneustic center, and the pneumotaxic center. The r
espiratory rhythmicity center sets the resting respiratory rate. The apneustic c
enter influences the respiratory rate by increasing the number of inspirations p
er minute. Its activity is countered by the pneumotaxic center, which inhibits i
nspiration. In times of increased demand, the pneumotaxic center decreases its i
nfluence, thereby increasing the respiratory rate. As the chest wall expands, me
chanical (stretch) receptors in the lungs send a signal to the apneustic center
via the vagus nerve to inhibit the inspiratory center, and expiration occurs. Th
is feedback loop, which combines neural and mechanical control, is called the He
ring-Breuer reflex. It is a protective mechanism that terminates inspiration, th
us preventing overexpansion of the lungs.
Answer is C
==== 114/115 Airway
A 44-year-old male was found unresponsive by his wife. According to the wife, he
had been drinking bourbon whiskey all day. He is unresponsive; has slow, shallo
w respirations; and a slow, weak pulse. You should:
A: ventilate him at a rate of 24 breaths/min with a bag-mask device.
B: promptly intubate his trachea and support his ventilations.
C: insert an oral airway and give oxygen via nonrebreathing mask.
D: insert a laryngeal mask airway to prevent aspiration if he vomits.
Reason:
Your patients airway is in immediate jeopardy! He is unresponsive, is unable to m
aintain his own airway, and has a stomach full of alcohol. Furthermore, the cent
ral nervous system depressant effects of alcohol are causing respiratory and car
diopulmonary insufficiency (eg, hypoventilation, bradycardia). First, you must p
rotect his airway from aspiration; this is most effectively accomplished by intu
bating his trachea. Slow, shallow respirations do not provide adequate minute vo
lume (his bradycardia is likely hypoxia-induced) and should be treated with posi
tive-pressure ventilation; a nonrebreathing mask is not sufficient. Assist the p
atients ventilations; provide 10 to 12 breaths/min with enough volume to produce
visible chest rise. Hyperventilationregardless of whether the patient is intubate
dhas been shown to be detrimental; it hyperinflates the lungs, which squeezes the
heart and reduces preload and subsequent cardiac output. Although the laryngeal
mask airway (LMA) has been shown to provide better positive-pressure ventilatio
n than the bag-mask device, it does not eliminate the risk of aspiration.
Answer is B
==== 115/115 Airway
A 56-year-old male presents with respiratory distress. He appears tired and is s
low to answer your questions. He is taking a series of quick breaths, followed b
y prolonged exhalation. On the basis of these clinical findings, you should:
A: intubate his trachea with the aid of pharmacologic agents.
B: provide some form of positive-pressure ventilation.
C: give oxygen via nonrebreathing mask and reassess him.
D: administer a beta-2 agonist via a hand-held nebulizer.
Your patient is not breathing adequately. Brief inhalations followed by prolonge
d exhalation will not produce adequate tidal volume; as a result, minute volume
will decrease. His tired appearance and delay in answering your questions are si
gns of decreased cerebral perfusion secondary to inadequate ventilation. You mus
t restore his tidal volume by providing some form of positive-pressure ventilato
ry assistance (eg, bag-mask ventilation). Patients with reduced tidal volume hav
e inadequate negative-pressure ventilation and need positive-pressure ventilatio
n. Passive oxygenation via nonrebreathing mask is appropriate for patients with
signs of hypoxemia whose tidal volume is adequate (eg, they are not breathing sh
allowly). Bronchospasm may be the underlying cause of the patients respiratory di
stress; however, because of his poor respiratory effort, he will not obtain the
full benefit of a beta-2 agonist via hand-held nebulizer. A small-volume nebuliz
er connected inline with the bag-mask device would be more effective. If the pat
ients mental status does not improve despite adequately performed ventilation ass
istance, intubation should be considered. Although the patient is tired and is s
low to answer your questions, he is still conscious. Therefore, pharmacologic ag
ents (ie, sedation, neuromuscular blockade) will be required if it is determined
that intubation is necessary.
Answer is B
.
EOF

Das könnte Ihnen auch gefallen